You are on page 1of 51

Downloaded From https://T.

me/TestSeries4Exams

VISIONIAS
www.visionias.in

Test Booklet Series

TEST BOOKLET

GENERAL STUDIES (P) 2019 – Test–2659


C
Time Allowed: Two Hours Maximum Marks: 200

INSTRUCTIONS
https://T.me/PDF4Exams

1. IMMEDIATELY AFTER THE COMMENCEMENT OF THE EXAMINATION, YOU SHOULD CHECK THAT THIS BOOKLET
DOES NOT HAVE ANY UNPRINTED OR TURN OR MISSING PAGES OR ITEMS, ETC. IF SO, GET IT REPLACED BY A
COMPLETE TEST BOOKLET.

2. ENCODE CLEARLY THE TEST BOOKLET SERIES A, B, C OR D AS THE CASE MAY BE IN THE APPROPRIATE PLACE IN
THE ANSWER SHEET.

3. You have to enter your Roll Number on the Test Booklet in the Box
provided alongside. Do NOT write anything else on the Test Booklet.

4. This Test Booklet contains 100items (Questions). Each item is printed in English. Each item comprises four
responses (answers). You will select the response which you want to mark on the Answer Sheet. In case you
feel that there is more than one correct response with you consider the best. In any case, choose ONLY ONE
response for each item.

5. You have to mark all your responses ONLY on the separate Answer Sheet provided. See direction in the
answers sheet.

6. All items carry equal marks. Attempt all items. Your total marks will depend only on the number of correct
responses marked by you in the answer sheet. For every incorrect response 1/3rdof the allotted marks will be
deducted.

7. Before you proceed to mark in the Answer sheet the response to various items in the Test booklet, you have to
fill in some particulars in the answer sheets as per instruction sent to you with your Admission Certificate.

8. After you have completed filling in all responses on the answer sheet and the examination has concluded, you
should hand over to Invigilator only the answer sheet. You are permitted to take away with you the Test
Booklet.

9. Sheet for rough work are appended in the Test Booklet at the end.

DO NOT OPEN THIS BOOKLET UNTIL YOU ARE ASKED TO DO SO


1 www.visionias.in ©Vision IAS

https://T.me/Civilsbuzz https://T.me/PDF4Exams
Downloaded From https://T.me/TestSeries4Exams

1. Which of the following are part of ‘basic 4. Consider the following statements regarding
structure’ of the Indian Constitution? the amending process of the Indian
1. Parliamentary system Constitution:
2. Welfare state 1. The President cannot veto a bill for
3. Federal character of the Constitution amendment of the Constitution.
4. Free and fair elections 2. Fundamental rights under Part III of the
Select the correct answer using the code Indian Constitution can be amended.
given below. Which of the statements given above is/are
(a) 1 and 3 only correct?
(b) 3 and 4 only
(a) 1 only
(c) 1, 2 and 4 only
(b) 2 only
(d) 1, 2, 3 and 4
(c) Both 1 and 2
(d) Neither 1 nor 2
https://T.me/PDF4Exams

2. Which of the following provisions depicts a


"unitary bias" in the Indian political system?
5. Freedom of speech and expression under
1. Emergency provisions
Article 19(1) (a) of the Constitution of India
2. Formation of new states
is subject to which of the following
3. All-India Services
reasonable restrictions?
Select the correct answer using the code
1. Sovereignty and integrity of India
given below.
2. Friendly relations with foreign states
(a) 1 only
3. Protection of interests of scheduled
(b) 1 and 2 only
tribes
(c) 2 and 3 only
4. Contempt of court
(d) 1, 2 and 3
Select the correct answer using the code

3. If the Central Government decides to settle a given below.

boundary dispute (excluding the exchange of (a) 3 and 4 only

territories) with China, it can be done by (b) 1, 2 and 3 only


(a) an executive order by giving effect to (c) 1, 2 and 4 only
the settlement. (d) 1 and 4 only
(b) amending the Constitution under Article
368. 6. The term, “Joint Comprehensive Plan of
(c) passing a law by an ordinary legislative Action” (JCPOA), is sometimes mentioned
process without amending the in the news in the context of the affairs of
Constitution. (a) Syria
(d) passing a law with the consent of the (b) Iran
states involved in the boundary (c) Iraq
settlement. (d) North Korea

2 www.visionias.in ©Vision IAS

https://T.me/Civilsbuzz https://T.me/PDF4Exams
Downloaded From https://T.me/TestSeries4Exams

7. Which of the following is not correct about 10. Which of the following provisions are
the European Union's General Data outside Part IV of the constitution?
Protection Regulation (GDPR)? 1. Freedom from attending religious
(a) It aims to protect European Union (EU) instruction in certain educational
citizens from data breaches. institutions.
(b) It adopts a rights-based approach. 2. To provide education for children until
they complete the age of six years.
(c) It applies only to Information
3. To protect and improve the natural
Technology companies.
environment.
(d) It provides for penalty in case of non-
4. No Person shall be deprived of his
compliance.
property save by authority of law.
Select the correct answer using the code
8. Which of the following acts seeks to protect below.
the Right against Exploitation under Article (a) 1 and 2 only
23? (b) 1, 2 and 4 only
https://T.me/PDF4Exams

1. Bonded Labour System (Abolition) Act, (c) 3 and 4 only


1976 (d) 1, 3 and 4 only
2. Minimum Wages Act, 1948
3. Untouchability Offences Act, 1955 11. The ownership of GST Network (GSTN) is
Select the correct answer using the code vested with the
given below. (a) GST Council
(a) 1 and 2 only (b) Reserve Bank of India
(b) 2 and 3 only (c) Ministry of Finance
(c) 3 only (d) Central and State governments
(d) 1, 2 and 3
12. Consider the following provision under
Article 20 of the Indian Constitution and the
9. Among all the committees of the Constituent
corresponding exceptions to them:
Assembly, the most important committee
Provision Exceptions
was the Drafting Committee since it was
1. No ex-post-facto : Preventive
tasked with the very important responsibility
law detention
of preparing a draft of the new Constitution.
2. No double : Proceedings
Along with the chairman Dr. B.R. Ambedkar jeopardy before
which among the following were the departmental
members of this committee? or
1. Syed Mohammad Saadullah administrative
2. Dr K M Munshi authorities
3. Dr. Rajendra Prasad 3. Protection against : Giving blood
4. B. PattabhiSitaramayya self-incrimination specimens
Select the correct answer using the code Which of the following pairs is/are correctly
given below. matched?
(a) 1 and 2 only (a) 1 and 3 only
(b) 2 and 4 only (b) 2 only
(c) 3 only
(c) 1 and 3 only
(d) 1, 2 and 3
(d) 1, 2, 3 and 4
3 www.visionias.in ©Vision IAS

https://T.me/Civilsbuzz https://T.me/PDF4Exams
Downloaded From https://T.me/TestSeries4Exams

13. The term „State‟ defined under Article 12 of 16. With reference to the
the Indian Constitution includes: PradhanMantriVaya VandanaYojana,
1. Legislative organs of Centre and states consider the following statements:
2. Executive organs of Centre and states 1. It is a pension scheme only for senior
citizens.
3. Local authorities
2. The scheme is being implemented
Select the correct answer using the code
through Life Insurance Corporation of
given below.
India (LIC).
(a) 1 and 2 only
3. Recently, the investment limit in it has
(b) 1 only been increased to Rs.15 lakhs.
(c) 2 only Which of the statements given above is/are
(d) 1, 2 and 3 correct?
(a) 1, 2 and 3
14. Consider the following statements: (b) 2 only
1. The State legislatures can initiate any (c) and 3 only
https://T.me/PDF4Exams

bill or proposal for amending the (d) 2 and 3 only

Constitution.
17. "Battlefield Tourism" is an initiative
2. The Constitution does not prescribe any
(a) initiative by India and China to declare
time frame within which the state
disputed territories as peace parks.
legislatures should ratify or reject an
(b) of Archeological Survey of India to
amendment submitted to them.
commemorate battle of Mahabharata in
3. Proclamation of Emergency does not Kurukshetra
require an amendment to the (c) of Ministry of Human Resource
Constitution. Development to generate national
Which of the statements given above is/are consciousness through Indian
correct? documentarieson Quit India movement.
(a) 1, 2 and 3 (d) to promote those historical sites where
(b) 1 and 2 only Indian soldiers have fought and died
over the ages as a tourism strategy.
(c) 2 and 3 only
(d) 1 and 3 only
18. Blockchain technology is often seen in news.
In this context consider the following
15. The idea of Fundamental Rights in Indian
statements:
Constitution was inspired by:
1. It is a digital public ledger that records
1. Government of India Act,1935 every transaction.
2. Nehru Report 2. Till now this technology can only be
3. United States Bill of Rights used in cruptocurrency.
Select the correct answer using the code Which of the statements given above is/are
given below. correct?
(a) 1 and 2 only (a) 1 only
(b) 2 and 3 only (b) 2 only
(c) Both 1 and 2
(c) 3 only
(d) Neither 1 nor 2
(d) 1, 2 and 3
4 www.visionias.in ©Vision IAS

https://T.me/Civilsbuzz https://T.me/PDF4Exams
Downloaded From https://T.me/TestSeries4Exams

19. Consider the following statements: 22. Consider the following statements regarding
1. The principle of natural justice ensures fundamental duties as provided under Indian
that no person shall be punished without
Constitution:
a fair hearing.
2. The expression 'natural justice' is not 1. Constitution authorizes Parliament for
mentioned in the Constitution of India. enforcement of the fundamental duties.
3. Article 32 of the Indian Constitution
2. While some fundamental duties are
provides constitutional remedies against
the violation of principle of natural applicable to citizens, few are applicable
justice. to foreigners and private organizations.
Which of the statements given above is/are Which of the statements given above is/are
correct?
(a) 1 only correct?
(b) 2 and 3 only (a) 1 only
(c) 1, 2 and 3 (b) 2 only
(d) 3 only
(c) Both 1 and 2
https://T.me/PDF4Exams

(d) Neither 1 nor 2


20. Recently the Cabinet approved 'National
Policy on Biofuels-2018' which categorised
biofuels into two broad categories i.e. 'basic' 23. Shyam has decided to renounce his Indian
and 'advanced' biofuels. In this regard
citizenship and acquire citizenship of
consider the following statements:
1. The basic biofuels consists of only first Canada. What will be the citizenship status
generation biofuels consisting of of his 5-year-old daughter?
bioethanol and biodiesel. (a) She will continue to hold Indian
2. The advanced biofuels consists of algae
citizenship.
based third generation biofuels.
Which of the statements given above is/are (b) She will lose her Indian citizenship and
correct? will not be allowed to acquire it in the
(a) 1 only
future.
(b) 2 only
(c) Both 1 and 2 (c) She will lose her Indian citizenship but
(d) Neither 1 nor 2 will have the right to acquire it when she
turns 18.
21. Which of the following rights falls under
(d) She will get both Indian and Canadian
„Right to religion‟ category under part III of
Indian constitution? citizenship.
1. Protection of language and culture of
minorities 24. The word "Political Justice" is mentioned in
2. Right to life and personal liberty
the Indian Constitution under
3. Freedom of conscience and free
profession, practice and propagation of (a) the Preamble only
religion (b) the Preamble and the Directive
Which of the statements given above is/are
Principles of State Policy
correct?
(a) 1 and 2 only (c) the Fundamental Rights and the
(b) 2 only Directive Principles of State Policy
(c) 3 only (d) the Preamble and Fundamental Rights
(d) 2 and 3 only
5 www.visionias.in ©Vision IAS

https://T.me/Civilsbuzz https://T.me/PDF4Exams
Downloaded From https://T.me/TestSeries4Exams

25. Consider the following statements regarding 28. The production linked subsidy for sugarcane
South Asia Wildlife Enforcement Network farmers in India is approved by the
(SAWEN):
(a) Commission for Agricultural Costs and
1. It is an inter-governmental wildlife law
Prices
enforcement support body aimed at
combating wildlife crime in South Asia. (b) Ministry of Consumer Affairs, Food and
2. It has been established under the aegis of Public Distribution
the South Asian Association for (c) Cabinet Committee on Economic Affairs
Regional Cooperation (SAARC). (d) Ministry of Finance
3. Its statute has not been adopted by India.
Which of the statements given above is/are
29. Which of the following are non-
correct?
(a) 1 only democratic forms of government?
(b) 1 and 2 only 1. Monarchy
(c) 2 and 3 only 2. Oligarchy
https://T.me/PDF4Exams

(d) 1, 2 and 3 3. Technocracy


4. Theocracy
26. ‘This river is often seen in the news due to
Select the correct answer using the code
inter-state water dispute. Noyyal is a major
tributary of this river and given below.
KrishnarajaSagara Dam was built on it. A (a) 1 and 2 only
famous port city flourished around this river (b) 2, 3 and 4 only
and which also served as the Capital of
(c) 1, 3 and 4 only
Chola kings’.
(d) 1, 2, 3 and 4
Which among the following river is best
described in the above passage?
(a) Godavari 30. Consider the following statements regarding
(b) Mahanadi SardarSwaran Singh committee, 1976 on
(c) Cauvery fundamental duties:
(d) Krishna
1. All the duties recommended by the
committee were included under
27. With reference to the Industrial and
Fundamental duties of Indian
Commercial Bank of China (ICBC) Credit
Suisse India Market Fund, consider the constitution.
following statements: 2. It provided for the implementation of
1. It is China‟s first India-dedicated fundamental duties to be kept out of the
publicly offered investment fund.
judicial review.
2. It will invest in infrastructure debt funds
Which of the following statements is/are
regulated by SEBI.
Which of the statements given above is/are correct?
correct? (a) 1 only
(a) 1 only (b) 2 only
(b) 2 only (c) Both 1 and 2
(c) Both 1 and 2
(d) Neither 1 nor 2
(d) Neither 1 nor 2
6 www.visionias.in ©Vision IAS

https://T.me/Civilsbuzz https://T.me/PDF4Exams
Downloaded From https://T.me/TestSeries4Exams

31. Which of the following statements regarding 34. A Presidential form of government is one
Government of India Act of 1858 where the :
is not correct?
1. Head of the state is popularly elected.
1. It created the new office of Secretary of
State. 2. Head of the state is not part of the
2. It abolished the Court of Directors and Cabinet
Board of Control. 3. Executive is responsible to the
3. The sovereign control over India was legislature.
divided between the British Crown and
Select the correct answer using the code
the East India Company.
Select the correct answer using the code given below.
given below. (a) 1 and 2 only
(a) 3 only (b) 3 only
(b) 2 and 3 only (c) 1 only
(c) 1 and 3 only
(d) 1 and 3 only
(d) 1, 2 and 3
https://T.me/PDF4Exams

32. With reference to the idea of liberty in the 35. Which of the following are fundamental
Indian constitution, consider the following duties as defined under Part IV-A of Indian
statements: constitution?
1. The Preamble seeks to secure the liberty
1. Spreading brotherhood among the
of thought and expression to all citizens
people
of India.
2. The ideals of liberty have been taken 2. Protecting the sovereignty, unity, and
from the Russian Revolution. integrity of India
3. All citizens of India enjoy absolute 3. Raising voice against injustice
liberty.
4. Supporting bonafide government
Select the correct answer using the code
given below. initiatives
(a) 1, 2 and 3 Select the correct answer using the code
(b) 1 and 3 only given below.
(c) 1 only (a) 1 and 2 only
(d) None
(b) 1 and 3 only
(c) 2 and 3 only
33. With reference to the Constituent Assembly,
consider the following statements: (d) 2 and 4 only
1. ManabendraNath Roy was the first to
put forward the idea of a Constituent 36. Guiyang and Dalian corridors recently seen
Assembly. in news, refer to:
2. It was based on the scheme formulated
(a) disputed territories between Japan and
by the Wavell Plan.
3. The representatives of British provinces North Korea.
were indirectly elected. (b) land to sea routes offered by China to
Which of the statements given above is/are Nepal for trade.
correct?
(c) Special Economic Zones established in
(a) 1 and 2 only
(b) 1 and 3 only India for Japanese investments.
(c) 3 only (d) Information Technology corridors
(d) 1, 2 and 3 launched by India in China.
7 www.visionias.in ©Vision IAS

https://T.me/Civilsbuzz https://T.me/PDF4Exams
Downloaded From https://T.me/TestSeries4Exams

37. Which of the following comes under the 41. Consider the following statement regarding
Right to Freedom? Fundamental Duties:
1. Right to assemble 1. They are not found in the constitutions
2. Right to association of socialist countries.
3. Right to movement 2. India was the first democratic country to
4. Right to profession codify fundamental duties as part of its
Select the correct answer using the code constitution.
given below. 3. USA, Canada and France have no
(a) 1 and 2 only specific provision for fundamental duties
(b) 1, 2 and 3 only as part of their constitution.
(c) 3 and 4 only Which of the following statements is/are
(d) 1, 2, 3 and 4 correct?
(a) 3 only
38. Which one of the following is a feature (b) 1 and 2 only
common to both the Indian Federation and (c) 1 and 3 only
(d) 2 and 3 only
https://T.me/PDF4Exams

the American Federation?


(a) Federation is the result of an agreement
between States. 42. With reference to the Constitutional
(b) Parliamentary form of government Monarchy, consider the following
(c) Dual citizenship statements:
(d) A federal supreme court to interpret the 1. It is a system of government where the
Constitution power of the monarch is limited by a
constitution.
39. The preamble of the Indian Constitution can 2. By being politically neutral, Monarch
be amended through: may provide political stability.
3. Japan is the only Asian nation having the
(a) Simple majority
constitutional monarchy.
(b) Special majority
Which of the statements given above are
(c) Simple Majority and consent of half of
correct?
the states
(a) 1 and 2 only
(d) Cannot be amended
(b) 1 and 3 only
(c) 2 and 3 only
40. The Parliament has exclusive authority to
(d) 1, 2 and 3
make a law of preventive detention for
reasons connected with:
43. Which of the following rights are
1. Security of a state
Fundamental Rights?
2. Foreign affairs
1. Right to hoist the National Flag by
3. Maintenance of public order
citizens
4. Maintenance of supplies and services
2. Right to sleep
essential to the community
3. Right to freedom of conscience
Select the correct answer using the code
Select the correct answer using the code
given below.
given below.
(a) 1 and 3 only
(a) 2 only
(b) 2 only
(b) 1 only
(c) 2 and 4 only
(c) 1 and 3 only
(d) 2, 3 and 4
(d) 1, 2 and 3
8 www.visionias.in ©Vision IAS

https://T.me/Civilsbuzz https://T.me/PDF4Exams
Downloaded From https://T.me/TestSeries4Exams

44. Which one of the following statements 47. Which of the following functions are
correctly describes the directive principles of performed by a Constitution?
state policy in the Indian Constitution? 1. To provide a set of basic rules that allow
(a) They are principles that lay down that for minimal coordination amongst
the goal of Indian polity is political members of a society.
democracy. 2. To enable the government to fulfill the
(b) They are the steps needed to prepare the aspirations of a society and create
masses for a communist struggle. conditions for a just society.
(c) They are moral precepts for the 3. To specify who has the power to make
decisions in a society.
authorities of the state.
4. To set some limits on what a
(d) They are recommendations for the
government can impose on its citizens.
application of ancient Indian wisdom to
Select the correct answer using the code
solve the problems of the modern world.
given below.
(a) 1 and 3 only
45. Consider the following statements regarding (b) 2 and 4 only
https://T.me/PDF4Exams

Community reserves in India: (c) 1, 3 and 4 only


1. They are declared under the purview of (d) 1, 2, 3 and 4
Forest Rights Act, 2006.
2. They can be declared in any community 48. With reference to the concept of 'equality',
land where a community has volunteered which of the following statements is correct?
to conserve wildlife and its habitat. (a) Political equality is sufficient to build
3. SingchungBugun community reserve in egalitarian and just society.
Arunachal Pradesh has recently been (b) Equality implies eliminating all forms of
awarded the Indian Biodiversity Award, differences among people.
2018. (c) Equality means all the person entitled
Which of the statements given above is/are same rights and opportunities to develop
correct? their skills and talents.
(a) 1 only (d) None of the above statements (a), (b)
(b) 2 only and (c) are correct.
(c) 2 and 3 only
(d) 1, 2 and 3 49. With reference to the Republican feature of
Indian polity, consider the following
46. With reference to the Press Council of India, statements:
consider the following statements: 1. India became a republic in 1947
1. It is a statutory body. immediately after passing of the the
2. It is usually chaired by the retired judge Indian Independence Act.
2. Republican feature is borrowed from the
of Supreme Court.
French Constitution.
3. It has the powers to take penal action
3. Head of the Indian state is directly
against the delinquent newspapers.
elected by the people.
Which of the statements given above is/are
Which of the statement given above is/are
correct?
correct?
(a) 1 and 2 only
(a) 1 and 2 only
(b) 3 only
(b) 2 only
(c) 1 and 3 only (c) 1 and 3 only
(d) 1, 2 and 3 (d) 1, 2 and 3
9 www.visionias.in ©Vision IAS

https://T.me/Civilsbuzz https://T.me/PDF4Exams
Downloaded From https://T.me/TestSeries4Exams

50. Rim of the Pacific (RIMPAC) 2018, the 53. Consider the following statements:
largest international maritime exercise in the 1. Rule of law states that society is
world was recently held. In this context, governed by law and this law applies
equally to all persons.
consider the following statements regarding
2. Supremacy of the constitution cannot
it:
exist unless the supremacy of the rule of
1. It was hosted by China.
law is established.
2. It was held in the South China Sea. Which of the statements given above is/are
3. India did not participate in this exercise. correct?
Select the correct answer using the code (a) 1 only
given below. (b) 2 only
(a) 1 and 2 only (c) Both 1 and 2
(b) 1, 2 and 3 (d) Neither 1 nor 2

(c) 2 only
54. The 86th constitutional amendment inserted/
(d) None
changed articles in which of the following
https://T.me/PDF4Exams

parts of the constitution?


51. Consider the following statements regarding 1. Part III
the declaration of „Right to Privacy‟ as a 2. Part IV
fundamental right: 3. Part IV-A
1. It is provided under Article 21 of the Select the correct answer using the code
Indian Constitution. given below.
2. It can be suspended during the (a) 1 only
(b) 1 and 2 only
imposition of National Emergency.
(c) 2 and 3 only
Which of the statements given above is/are
(d) 1, 2 and 3
correct?
(a) 1 only
55. Consider the following pairs:
(b) 2 only Features of the Source
(c) Both 1 and 2 Indian Constitution
(d) Neither 1 nor 2 1. Advisory Jurisdiction : Canada
of the Supreme Court
52. Consider the following statements regarding 2. Removal of High : USA
Non-Resident Indians (NRI): Court Judges
3. Joint sitting of the : Australia
1. An NRI is one who is an Indian citizen
two Houses
but ordinarily residing outside India.
of Parliament
2. NRIs have to apply for visa whenever 4. Nomination of : Ireland
they are traveling to India. members to
3. They do not enjoy voting rights in India. RajyaSabha
Which of the statements given above is/are Which of the pairs given above are correctly
correct? matched?
(a) 1 and 2 only (a) 2 and 4 only
(b) 1, 3 and 4 only
(b) 2 and 3 only
(c) 1, 2 and 3 only
(c) 1 only
(d) 1, 2, 3 and 4
(d) 1, 2 and 3
10 www.visionias.in ©Vision IAS

https://T.me/Civilsbuzz https://T.me/PDF4Exams
Downloaded From https://T.me/TestSeries4Exams

56. The PM recently inaugurated the state-run 59. Directive Principles of State Policy were
NHPC Ltd‟s Kishanganga hydro power made non-justiciable in nature because of:
project. In this context, consider the 1. lack of financial resources for
following statements: implementation.
1. It is located in Himachal Pradesh. 2. the belief of the Constitution-makers on
2. It envisages diversion of water from the public opinion as the ultimate sanction
Kishanganga River to a power plant in
for the fulfilment of these principles.
the Jhelum River basin through an
Select the correct answer using the code
underground tunnel.
given below.
Which of the statements given above is/are
(a) 1 only
correct?
(b) 2 only
(a) 1 only
(c) Both 1 and 2
(b) 2 only
(d) Neither 1 nor 2
(c) Both 1 and 2
https://T.me/PDF4Exams

(d) Neither 1 nor 2


60. The Supreme Court can issue a writ

57. The Network for Spectrum Project, being of mandamus under Article 32 to direct:

implemented by the Bharat Sanchar Nigam 1. the central government to introduce an


Limited (BSNL), is meant to amendment bill in the LokSabha.
(a) lay alternative communications network 2. the state's government to grant remission
for the defence services. of sentence.
(b) establish a high speed communications Select the correct answer using the code
network for commercial usage. given below.
(c) expand high speed data services in (a) 1 only
disaster prone regions. (b) 2 only
(d) overhaul the urban digital infrastructure (c) Both 1 and 2
for facilitating setting up of Intelligent
(d) Neither 1 nor 2
Transportation Systems.

61. Which among the following schemes have


58. Constitution of India ensures the protection
been subsumed under the recently launched
of women's rights and welfare through
SamagraShiksha Scheme?
which of the following?
1. SarvaShikshaAbhiyan
1. Article 14
2. RashtriyaMadhyamikShikshaAbhiyan
2. Article 16
3. RashtriyaUchchatarShikshaAbhiyan
3. Article 39(a)
Select the correct answer using the code Which of the statements given above are

given below. correct?

(a) 1, 2 and 3 (a) 1 and 2 only


(b) 1 and 2 only (b) 2 and 3 only
(c) 2 and 3 only (c) 1 and 3 only
(d) 1 only (d) 1, 2 and 3

11 www.visionias.in ©Vision IAS

https://T.me/Civilsbuzz https://T.me/PDF4Exams
Downloaded From https://T.me/TestSeries4Exams

62. Consider the following statements: 65. Which among the following statements
1. Foreign ambassadors and diplomats regarding Article 21 of the Constitution of
working in India enjoy immunity from
India is/are correct?
criminal and civil proceedings.
2. Foreigners holding any office of profit 1. The Right to personal liberty under
under government of India cannot accept Article 21 as a fundamental right has
emoluments from any foreign state been provided against the state only.
without the consent of the Prime
2. It is violated when accused dies in the
Minister.
Which of the statements given above is/are police custody due to torture by the
correct? police.
(a) 1 only Select the correct answer using the code
(b) 2 only
given below.
(c) Both 1 and 2
(d) Neither 1 nor 2 (a) 1 only
(b) 2 only
https://T.me/PDF4Exams

63. Consider the following statements (c) Both 1 and 2


regarding DNA profiling :
(d) Neither 1 nor 2
1. For DNA profiling, the DNA can only
be obtained from the skin and saliva of
person. 66. Indian constitution provided provision of
2. It can be used for determining parenting. three-tier Government, what does it implies?
3. Recently, Andhra Pradesh has become (a) Seperation of power between executive,
India‟s first state that launched DNA
judiciary and legislature.
profiling for criminals.
Which of the statements given above is/are (b) Distribution of powers between centre,
correct? state and local level.
(a) 1 only
(c) Presence of constitutional post of
(b) 2 only
President, Vice President and Governor.
(c) 2 and 3 only
(d) 1, 2 and 3 (d) None of the above

64. Article 17 of Indian Constitution provides 67. Which of the following committee(s) is/are
for abolition of untouchability. Consider the
related to state reorganization post-
following statements in this context:
independence?
1. It is the only fundamental right available
against State as well as private 1. Fazl Ali Commission
individuals. 2. Ashok Mehta Committee
2. It has defined the term 'untouchability'.
3. Dhar Committee
3. It provides for Constitutional remedies
for social boycott Select the correct answer using the code
Which of the statements given above is/are given below.
correct? (a) 1 only
(a) 1 only
(b) 1 and 3 only
(b) 2 and 3 only
(c) 3 only (c) 2 and 3 only
(d) None (d) 1 and 2 only
12 www.visionias.in ©Vision IAS

https://T.me/Civilsbuzz https://T.me/PDF4Exams
Downloaded From https://T.me/TestSeries4Exams

68. The term Net Stable Funding 71. What do we mean when we say
Ratio sometimes seen in the news, is related Fundamental Rights are not absolute?
to (a) They are subject to reasonable
(a) IMF quota reforms restrictions.
(b) Carbon trading norms (b) They can not be repealed or modified by
(c) BASEL Norms amending the Constitution by the State
(d) WTO Government.
(c) They are available to both Citizens and
69. In which of the following instances can the non citizens.
High Courts be approached for violation of (d) None of the statements (a), (b) and (c)
fundamental right? are correct.
1. Caste panchayat declaring punishment to
a young couple for inter-caste marriage. 72. Which of the following Directive Principles
2. Acquisition of land without following was/were added by
https://T.me/PDF4Exams

nd
the procedure prescribed by law. 42 Constitutional Amendment Act of
3. Government banning the religious 1976?
practice of human sacrifice. 1. To promote equal justice and to provide
Select the correct answer using the code free legal aid to the poor.
given below. 2. To minimize inequalities in income,
(a) 1 only status, facilities and opportunities.
(b) 2 only 3. To protect and improve the environment
(c) 3 only and to safeguard forests and wildlife.
(d) 1 and 2 only Select the correct answer using the code
given below.
70. With reference to the Cultural and (a) 1 and 2 only
Educational Rights of minorities, consider (b) 3 only
the following statements: (c) 1 and 3 only
1. Only the religious and linguistic (d) 1, 2 and 3
minorities have the right to establish and
administer educational institutions. 73. Which of the following is/are devices of
2. Only the rights of religious and direct democracy?
linguistic minorities have been 1. Recall
constitutionally safeguarded to preserve 2. Initiative
their distinct culture. 3. Public Interest Litigation
Which of the statements given above is/are Select the correct answer using the code
correct? given below.
(a) 1 only (a) 1 only
(b) 2 only (b) 1 and 2 only
(c) Both 1 and 2 (c) 3 only
(d) Neither 1 nor 2 (d) 1, 2 and 3

13 www.visionias.in ©Vision IAS

https://T.me/Civilsbuzz https://T.me/PDF4Exams
Downloaded From https://T.me/TestSeries4Exams

74. What is/are the purpose of One Stop Centre 77. Green Good Deeds recently seen in news,
(OSC) Scheme launched by the government? refers to a
1. Issue of birth certificates (a) global campaign launched by
2. Support to women affected by violence
Greenpeace to spread awareness about
3. Vaccination of children
green technologies.
4. Registration for jobs
(b) scheme launched by Indian Railways to
Select the correct answer using the code
promote green and clean railways.
given below.
(a) 1 and 4 only (c) societal movement to protect

(b) 2 only environment and promote good living.


(c) 2 and 3 only (d) tax exemptions provided by the
(d) 1, 2, 3 and 4 government for those entities engaed in
green technologies.
75. Consider the following legislation enacted
https://T.me/PDF4Exams

by a Parliament of India: 78. Which of the following were introduced by


1. Protection of the Civil Rights Act, 1976
the Government of India Act of 1935 in the
2. Child Labour Prohibition and Regulation
British provinces?
Act, 1986
1. Dyarchy
3. National Food Security Act, 2013
Which of the following is/are 2. Responsible government

operationalized/implemented by the above- 3. Bicameralism


mentioned legislation? Select the correct answer using the code
(a) Directive Principles of State Policy given below.
(b) Fundamental Rights (a) 1 and 2 only
(c) Directives outside part IV of Indian (b) 1 and 3 only
Constitution
(c) 2 and 3 only
(d) Both a and b
(d) 1, 2 and 3

76. The Second Schedule deals with the


79. Which of the following is/are features of
emoluments, allowances and privileges of
Indian parliamentary system?
1. President
2. Members of Parliament 1. Majority party rule
3. Comptroller and Auditor General of 2. Presence of real and nominal Executive
India 3. Membership of the ministers in the
4. Speaker and Deputy Speaker of legislature
LokSabha Select the correct answer using the code
Select the correct answer using the code
given below.
given below.
(a) 1, 2 and 3
(a) 1, 2 and 3 only
(b) 1 and 2 only
(b) 1, 3 and 4 only
(c) 2 and 3 only
(c) 1, 2 and 4 only
(d) 3 and 4 only (d) 1 only

14 www.visionias.in ©Vision IAS

https://T.me/Civilsbuzz https://T.me/PDF4Exams
Downloaded From https://T.me/TestSeries4Exams

80. If a person has voluntarily acquired the 83. Consider the following pairs regarding

citizenship of a foreign state, which among Constitutional amendment act and their
objective:
the following is/are the likely outcome/s?
Constitutional Objective
1. Disqualification from being elected as a
Amendment Act
member of the Indian Parliament 1. 24th : Parliament can
2. Non-eligibility to hold the office of Amendment amend

President of India Fundamental


Rights
Select the correct answer using the code
2. 42nd : Introduction of
given below.
Amendment Fundamental
(a) 1 only Duties
(b) 2 only 3. 61st : Reducing the
https://T.me/PDF4Exams

(c) Both 1 and 2 Amendment age of voting to


18 years
(d) Neither 1 nor 2
Which of the pairs given above are correctly
matched?
81. Recently with the initiative of Indian
(a) 1 and 2 only
Railways, Cheriyal scroll paintings was in
(b) 2 and 3 only
news. This painting is a well known art form (c) 1 and 3 only
of which of the following State? (d) 1, 2 and 3

(a) Karnataka
84. Consider the following statements regarding
(b) Tamil Nadu
the recently set up National Water
(c) Telengana
Informatics Centre:
(d) Nagaland 1. It has been constituted as a Special
Purpose Vehicle with the participation of
82. Which among the following important government and private sector.

constitutional amendments was added to the 2. It would act as repository of nation-wide


water resources data.
Constitution earliest?
3. It will replace the existing India Water
(a) Inclusion of Bodo, Dongri, Maithili and
Resources Information System (India-
Santhali in Eighth Schedule. WRIS).
(b) State can make any provision for Which of the statements given above is/are

reservation in matters of promotions for correct?


(a) 1 and 3 only
SC and ST‟s in state services.
(b) 2 only
(c) Right to form cooperatives societies.
(c) 3 only
(d) Right to Education. (d) 1, 2 and 3
15 www.visionias.in ©Vision IAS

https://T.me/Civilsbuzz https://T.me/PDF4Exams
Downloaded From https://T.me/TestSeries4Exams

85. If the State legislature passes a bill 88. Consider the following pairs:
mandating concession in fees for education Country Form of
to its residents, then: Government
(a) the Governor may ratify the bill. 1. France : Semi-presidential
2. Saudi Arabia : Absolute Monarchy
(b) the Governor may reject the bill citing
3. United : Parliamentary
the reason that the bill is against Article
Kingdom Republic
15 of the Indian Constitution.
Which of the pairs given above are correctly
(c) it will be mandatory for the Governor to
matched?
reserve the bill for the President.
(a) 1 and 2 only
(d) the Parliament will have to pass a law (b) 2 and 3 only
mandating the same to make the (c) 1 and 3 only
provision enforceable. (d) 1, 2 and 3
https://T.me/PDF4Exams

86. Which of the following could be possible 89. Consider the following statements regarding
reasons for the inclusion of fundamental recently launched Green Skill Development
duties in the Indian Constitution? Programme (GSPD):
1. Making citizens conscious of their 1. It is aimed at imparting skills which are
responsibility required to adapt services and products
to climate change.
2. Strengthening of democracy
2. It is an initiative under the National Skill
3. Promoting socio-economic equality
Development Mission.
Select the correct answer using the code
Which of the statements given above is/are
given below.
correct?
(a) 1 only
(a) 1 only
(b) 1 and 2 only (b) 2 only
(c) 3 only (c) Both 1 and 2
(d) 1, 2 and 3 (d) Neither 1 nor 2

87. Which among the following provisions 90. Government of India can increase the
directly seek to promote the feeling of territorial extent of 'Territory of India' by:
fraternity? 1. Declaring a Union Territory as a state.
1. Preamble 2. Acquiring new areas through conquest

2. Fundamental Duties or subjugation.


3. Accepting land as a gift from another
3. Directive Principles of State Policy
country.
Select the correct answer using the code
Select the correct answer using the code
given below.
given below.
(a) 2 only
(a) 1 and 2 only
(b) 1 and 2 only
(b) 1 and 3 only
(c) 3 only (c) 2 and 3 only
(d) 1, 2 and 3 (d) 1, 2 and 3

16 www.visionias.in ©Vision IAS

https://T.me/Civilsbuzz https://T.me/PDF4Exams
Downloaded From https://T.me/TestSeries4Exams

91. Multi-sectoral Development Programme was 94. Who among the following can introduce a

recently restructured to PradhanMantri Jan bill to amend the Constitution of India?


1. A minister who is not a part of the
VikasKaryakram (PMJVK). It's objective is
Parliament
to:
2. Leader of Opposition
(a) Provide better educational facilities to
3. Attorney General
the tribal communities Select the correct answer using the code

(b) Provide better socio-economic given below.

infrastructure facilities to the minority (a) 1 and 3 only


(b) 1 only
communities
(c) 2 and 3 only
(c) Promote social harmony in society
(d) 1 and 2 only
https://T.me/PDF4Exams

through interstate student exchange

programmes 95. Which of the following methods are


available to any person for acquiring
(d) Reduce poverty levels to 15% by 2022
citizenship of India under the Citizenship
Act of 1955?
92. Suva expert dialogue, often seen in news, is
1. By naturalization
aimed at addressing issue of
2. By incorporation of the territory
(a) rising global inequality
3. By descent
(b) terrorism at the global level. Select the correct answer using the code
(c) impact of trade wars on developing given below.

countries. (a) 1 only


(b) 1 and 3 only
(d) loss and damage associated owing to
(c) 2 only
climate change.
(d) 1, 2 and 3

93. Which one of the following is the best 96. The Preamble of India reveals:

description of „S-400 Triumf‟, that was in


(a) Source of authority of the Constitution.
the news recently?
(b) Objectives of the Constitution.
(a) nuclear attack submarine

(b) generic medicine to fight ovarian cancer (c) Date of adoption of the Constitution.

(c) air defence system


(d) All statements (a), (b) and (c) are
(d) genetic variety of rice for diabetics correct.

17 www.visionias.in ©Vision IAS

https://T.me/Civilsbuzz https://T.me/PDF4Exams
Downloaded From https://T.me/TestSeries4Exams

97. Which of the following statements is/are 99. Consider the following statements:
correct about the Regional Anti-Terrorism
1. The area or boundary of any state can by
Structure (RATS) of the Shanghai
Cooperation Organisation (SCO)? altered by Parliament.

1. It is a permanent body of the SCO. 2. The name of the state can be altered by
2. All member countries of SCO are part of
its own State Legislature.
it.
3. It is headquartered at Tashkent. Which of the statements given above is/are

Select the correct answer using the code correct?


given below.
(a) 1 only
(a) 1, 2 and 3
(b) 2 only (b) 2 only
(c) 1 and 3 only
(c) Both 1 and 2
https://T.me/PDF4Exams

(d) 3 only
(d) Neither 1 nor 2

98. With reference to recently concluded


SwachhSurvekshan 2018, consider the 100. Consider the following situations:
following statements:
1. Carrying of Kirpans by Sikhs.
1. NITI Aayog is responsible for carrying
out the assessment. 2. Establishment of a madrasa by Maulvi.

2. For score calculation, collection and 3. Admission of a scheduled caste student


transportation of Municipal solid waste
in a state-funded primary school.
management is assigned the maximum
weight of 40%. Which of the above provisions are protected
3. Jharkhand has been adjudged as the by the fundamental rights, as enshrined in
best-performing state.
the Indian constitution?
Which of the statements given above is/are
correct? (a) 2 and 3 only
(a) 1 and 2 only
(b) 1 and 2 only
(b) 1 only
(c) 1 and 3 only
(c) 3 only
(d) 2 and 3 only (d) 1, 2 and 3

Copyright © by Vision IAS


All rights are reserved. No part of this document may be reproduced, stored in a retrieval system or transmitted in any
form or by any means, electronic, mechanical, photocopying, recording or otherwise, without prior permission of
Vision IAS

18 www.visionias.in ©Vision IAS

https://T.me/Civilsbuzz https://T.me/PDF4Exams
Downloaded From:- https://T.me/TestSeries4Exams

VISIONIAS
www.visionias.in

ANSWERS & EXPLANATION


GENERAL STUIDES (P) TEST – 2659 (2019)

Q 1.D
 All the statements are correct.
 The validity of the Twenty-fifth Amendment was challenged in Kesavananda Bharati v. the State of
Kerala along with the Twenty-fourth and Twenty-ninth Amendments. The Court overruled Golaknath
case judgment and declared that Article 368 did not enable Parliament to alter the basic structure or
https://T.me/PDF4Exams

framework of the Constitution. The Judges made no attempt to define the basic structure of the
Constitution in clear terms. Following are the parts of the basic structure of the constitution:
o Supremacy of the Constitution
o A Republican and democratic form of Government
o The secular character of the Constitution
o Separation of powers between the legislature, the executive and the judiciary
o Federal character of the Constitution
o Unity and integrity of the nation
o Welfare state (socio-economic justice)- Bhim Singhji Case (1981)
o Judicial review
o Freedom and dignity of the individual
o Parliamentary system
o Rule of law
o Harmony and balance between Fundamental Rights and Directive Principles
o Principle of equality
o Free and fair elections- Indira Gandhi case (1975) and Kihoto Hollohon Case (1993) (popularly
known as Defection case)
o Independence of Judiciary
o Limited power of Parliament to amend the Constitution
o Effective access to justice
o Principles (or essence) underlying fundamental rights
o Powers of the Supreme Court under Articles 32, 136, 141 and 1426
o Powers of the High Court‘s under Articles 226 and 227

Q 2.D
 Some of the provisions depict a "unitary bias" in the Indian political system:
o The Constitution has certain very powerful emergency provisions, which can turn our federal polity into
a highly centralized system once emergency is declared. During an emergency, power becomes lawfully
centralized. Parliament also assumes the power to make laws on subjects within the jurisdiction of the
States.

1 www.visionias.in ©Vision IAS

https://T.me/Civilsbuzz https://T.me/PDF4Exams
Downloaded From:- https://T.me/TestSeries4Exams

o The very existence of a State including its territorial integrity is in the hands of Parliament. The
Parliament is empowered to ‗form a new State by separation of territory from any State or by
uniting two or more States…‘. It can also alter the boundary of any State or even its name. The
Constitution provides for some safeguards by way of securing the view of the concerned State legislature.
o The all-India services are common to the entire territory of India and officers chosen for these services
serve in the administration of the States. Thus, an IAS officer who becomes the collector or an IPS officer
who serves as the Commissioner of Police, are under the control of the central government. States cannot
take disciplinary action nor can they remove these officers from service.
 Hence, all the statements are correct.

Q 3.A
 The Supreme Court has held that the power of Parliament to diminish the area of a state (under Article 3)
does not cover cession of Indian territory to a foreign country. Hence, Indian territory can be ceded to a
foreign state only by amending the Constitution under Article 368.
 On the other hand, the Supreme Court in 1969 ruled that settlement of a boundary dispute between
India and another country does not require a constitutional amendment. It can be done by
executive action as it does not involve a cession of Indian territory to a foreign country.
https://T.me/PDF4Exams

Q 4.C
 Statement 1 is correct: It is obligatory upon the President to give his assent to a bill for amendment of
the Constitution, when it is presented to him after the passage by the legislature. The President‘s power to
veto a bill for amendment of the Constitution has been taken away by the 24th Constitution amendment
act.
 Statement 2 is correct: Fundamental rights under Part III of Indian Constitution can be amended by an
Act passed under Article 368 and the validity of a Constitution amendment act cannot be questioned on
the ground that the act invades or encroaches upon any Fundamental right. However, there are
certain Basic features of the Indian Constitution, which cannot be altered by Parliament in exercise of the
power to amend Indian Constitution under Article 368.

Q 5.C
 The Constitution of India guarantees various fundamental rights to its citizens. One such important right is
right to freedom under Article 19.
 Article 19(1)(a)guarantees that all the citizens have the right to freedom of speech and expression. This
right is available only to the citizens of India and not available to any person who is not a citizen of India
i.e. foreign nationals.
 No freedom can be absolute or completely unrestricted.
 Article 19(2) specifies the grounds to which reasonable restrictions on the freedom of speech and
expression can be imposed under Article 19(2) reasonable restrictions can be imposed on freedom of
speech and expression on the following ground:
o in the interest of the security of State.
o for the maintenance of good relations between India and foreign State.
o Public order which is more than the ordinary maintenance of law and order and also includes public
peace, safety and tranquility.
o Decency or Morality
o Contempt of Court
o Defamation or Incitement to an offense
o Sovereignty and Integrity of India
2 www.visionias.in ©Vision IAS

https://T.me/Civilsbuzz https://T.me/PDF4Exams
Downloaded From:- https://T.me/TestSeries4Exams

Q 6.B
 The Joint Comprehensive Plan of Action (JCPOA), commonly known as the Iran deal, was signed
on July 14, 2015 between Iran, the U.S., China, France, Russia, the U.K., Germany and the
European Union.
 Under this deal, Iran agreed not to build any more heavy water facilities, eliminate its stockpile or
medium-enriched uranium, cut its stockpile of low-enriched uranium, and reduce the number of its gas
centrifuges.
 Other nuclear facilities in Iran were to be converted into non-nuclear facilities. The International Atomic
Energy Agency would have access to all facilities in Iran to monitor and verify if the West Asian country
was acceding to the terms of the agreement.
 In return, Iran was to recover assets worth $100 billion frozen in overseas banks, and sanctions on the
country by the U.S., the U.N. and the E.U. would be lifted.
 However, in May 2018, the US pulled out of the JCPOA on the pretext that it failed to deal with the threat
of Iran‘s missile programme and did not include a strong enough mechanism for inspections and
verification.

Q 7.C
https://T.me/PDF4Exams

 General Data Protection Regulation (GDPR) aims to protect all European Union(EU) citizens from
data breaches. The regulation, which was approved by the EU Parliament in April 2016 after about four
years of preparation and debate, came into effect on May 25, 2018. Hence statement (a) is correct.
 The GDPR adopts a rights-based, consent-driven approach towards protecting the data of natural
persons. The GDPR makes it clear that it protects the fundamental right to protection of data of natural
persons, and goes on to establish, amongst others, rights to data access, rectification, erasure, restriction,
portability and objection. Hence statement (b) is correct.
 It is not just the Information Technology(IT) and Information Technology enabled Services(ITeS)
companies, but firms across sectors and industries that need to be GDPR-compliant. Hence
statement (c) is not correct.
 Also, any organization providing goods and services in the EU but functioning outside EU comes under
GDPR.
 The GDPR includes appointing a data protection officer in organizations, the concept of privacy by design
(encryption) and by default (processing the minimum amount of data), and new consent rules which
require consent for different activities from different stakeholders, including employees and customers. It
also provides for penalties of up to €20 million or 4% of a company‘s global revenue for non-
compliance. Hence statement (d) is correct.
 Other Information - Exemptions/ restrictions: Following cases are not covered by the regulation:
 Lawful interception, national security, military, police, justice
 Statistical and scientific analysis
 Deceased persons, subject to national legislation
 Employer-employee relationships (covered as per a separate law)
 Processing of personal data by a natural person in the course of a purely personal or household
activity
 Conversely, an entity has to be engaged in "economic activity" (as per EU laws) to fall under GDPR.

Q 8.A
 Right Against Exploitation (Article 23 & 24) is a fundamental right enshrined in the Constitution of
India.
 Article 23 (Prohibition of Traffic in Human Beings and Forced Labour) prohibits traffic in human beings,
begar (forced labor) and other similar forms of forced labor. Any contravention of this provision shall be
an offense punishable in accordance with law. This right is available to both citizens and non-citizens. It
protects the individual not only against the State but also against private persons.
3 www.visionias.in ©Vision IAS

https://T.me/Civilsbuzz https://T.me/PDF4Exams
Downloaded From:- https://T.me/TestSeries4Exams

 In this regard, various acts have been enacted such as


 Bonded Labour System (Abolition) Act, 1976. Hence option 1 is correct.
 Minimum Wages Act, 1948. Hence option 2 is correct.
 Contract Labour Act, 1970. Equal Remuneration Act, 1976
 Article 24 (Prohibition of Employment of Children in Factories, etc) prohibits the employment of
children below the age of 14 years in any factory, mine or other hazardous activities like construction
work or railway. But it does not prohibit their employment in any harmless or innocent work.
 The Untouchability (offences) act of 1955 was passed by the parliament of India to spell out article 17 of
the constitution. Article 17 abolishes ‗untouchability‘ and forbids its practice in any form. The
enforcement of any disability arising out of untouchability shall be an offense punishable in accordance
with law. Hence option 3 is not correct.
 In 1976, the Untouchability (Offences ) Act, 1955 has been comprehensively amended and renamed as the
Protection of Civil Rights Act, 1955 to enlarge the scope and make penal provisions more stringent. The
act defines civil right as any right accruing to a person by reason of the abolition of untouchability by
Article 17 of the Constitution.

Q 9.A
https://T.me/PDF4Exams

 Among all the committees of the Constituent Assembly, the most important committee was the Drafting
Committee set up on August 29, 1947. It was this committee that was entrusted with the task of preparing
a draft of the new Constitution. It consisted of seven members. They were:
 Dr B R Ambedkar (Chairman)
 N Gopalaswamy Ayyangar
 Alladi Krishnaswamy Ayyar
 Dr K M Munshi
 Syed Mohammad Saadullah
 N Madhava Rau (He replaced B L Mitter who resigned due to ill-health)
 T T Krishnamachari (He replaced D P Khaitan who died in 1948)

Q 10.D
 Statement 1 is a fundamental right (Article 28) under part III.
 Statement 2 is directive principle (Article 45) under part IV.
 Statement 3 is a fundamental duty (Article 51-A) under part IV-A.
 Statement 4 is a constitutional right (Article 300-A) under part XII.
 Hence option (d) is the correct answer.
 Following Rights are mentioned in the constitution outside part III, this makes them constitutional but not
fundamental rights and part of directive principles. These Rights are:
 No tax shall be levied or collected except by authority of law. (Article 265)
 No person shall be deprived of his property save by authority of law (Article 300-A).
 Trade, Commerce and intercourse throughout the territory of India shall be free (Article 301)
 Adult Suffrage (Article 326)

Q 11.D
o The Goods and Services Tax Network - Special Purpose Vehicle (GSTN-SPV) was created as a private
limited, not-for-profit company with an objective to provide shared IT infrastructure and services to
Centre and States Governments, tax payers and other stakeholders for implementation of Goods and
Services Tax (GST) in the country.
o Majority of the GST processes including registration, filing of returns, payment of taxes, processing of
refunds is IT driven and GSTN is handling large-scale invoice level data of lakhs of business entities
including data relating to exports and imports. Considering the nature of ‗state‘ function performed by
GSTN, Council felt that GSTN be converted into be a fully owned government Company.
4 www.visionias.in ©Vision IAS

https://T.me/Civilsbuzz https://T.me/PDF4Exams
Downloaded From:- https://T.me/TestSeries4Exams

o The Council decided in May, 2018 that the entire 51 per cent of equity held by the non-
governmental institutions in GSTN amounting to Rs 5.1 crore would be acquired equally by the
Central and State Governments.
o Earlier, The GST Network was 24.5 per cent owned by the Central government, 24.5 per cent held by
state governments collectively, and the remaining 51 per cent is with HDFC Ltd, HDFC Bank Ltd, ICICI
Bank Ltd, NSE Strategic Investment Co and LIC Housing Finance Ltd.

Q 12.D
o Pair 1 is correctly matched: Article 20 provides for no ex-post-facto law. Hoewever, this limitation on
enactment of ex-post-facto law is only on criminal laws and not on civil laws or tax laws. Also, protection
under this provision can‘t be claimed in case of preventive detention or demanding security from a person.
o Pair 2 is correctly matched: The provision of no double jeopardy under Article 20 is available only in
proceeding before a court of law or a judicial tribunal. It is not available before departmental or
administrative authorities as they are not of judicial nature.
o Pair 3 is correctly matched: The protection against self-incrimination under Article 20 extends to both
oral evidence and documentary. However, it doesn‘t cover -:
https://T.me/PDF4Exams

 Compulsory production of material objects;


 Compulsion to give thumb impression, specimen signature, blood specimens; and
 Compulsory exhibition of the body.

Q 13.D
o Article 12 defines the term ‗State‘ as used in different Articles of Part III of the Constitution. It says
that unless the context otherwise requires the term ‗State‘ includes the following-
 The Government and Parliament of India, i.e., Executive and Legislature of the Union.
 The Government and Legislature of each State, i.e., Executive and Legislature of State.
 All local and other authorities within the territory of India.
 All local and other authorities under the control of the Government of India.
o According to Article 36, the term ‗State‘ in Part IV has the same meaning as in Part III dealing with
Fundamental Rights. Hence, the correct answer is (d).
o In one of its latest observations, the Supreme Court has held that judiciary can be considered as a ‗State‘
as far as its rule-making power is concerned, but it would not be considered so when it exercises its
judicial powers.

Q 14.C
o Statement 1 is not correct: The power to initiate an amendment to the Constitution lies with the
Parliament. Hence, unlike in the USA, the state legislatures cannot initiate any bill or proposal for
amending the Constitution except in one case, that is, passing a resolution requesting the
Parliament for the creation or abolition of legislative councils in the states, but creation or abolition
of legislative council are not considered an amendment to the Constitution. Here also, the Parliament
can either approve or disapprove such a resolution or may not take any action on it.
o Statement 2 is correct: The Constitution does not prescribe the time frame within which the state
legislatures should ratify or reject an amendment submitted to them. Also, it is silent on the issue
whether the states can withdraw their approval after according the same.
o Statement 3 is correct: During an Emergency, the Central government becomes all powerful and the
states go into the total control of the Centre. It converts the federal structure into a unitary one
without a formal amendment of the Constitution.

5 www.visionias.in ©Vision IAS

https://T.me/Civilsbuzz https://T.me/PDF4Exams
Downloaded From:- https://T.me/TestSeries4Exams

Q 15.B
o The rights of the Nehru Report were a close precursor of the Fundamental Rights of the
Constitution. It proposed 19 fundamental rights including rights for women, the right to form unions and
universal adult suffrage etc.
o The Constitution framers while considering the provisions on Fundamental Rights had consulted the
working Constitutions of a number of countries. Thus the development of constitutionally guaranteed
fundamental human rights in India was inspired by historical examplessuch as England's Bill of
Rights (1689), the United States Bill of Rights (approved on 17 September 1787, final ratification on
15 December 1791) and France's Declaration of the Rights of Man (created during the revolution of 1789,
and ratified on 26 August 1789).
o The government of India Act of 1935 did not propose for Fundamental Rights.
o Hence only statements 2 and 3 are correct.

Q 16.A
o Statements 1 and 2 are correct: Pradhan Mantri Vaya Vandana Yojana (PMVVY) is a Pension
Scheme announced by the Government of India exclusively for the senior citizens aged 60 years and
https://T.me/PDF4Exams

above which is available from 4th May, 2017 to 31st March, 2020.The PMVVY is being
implemented through Life Insurance Corporation of India (LIC) to provide social security during old
age and protect elderly persons aged 60 years and above against a future fall in their interest income due
to uncertain market conditions.
o The scheme provides an assured pension based on a guaranteed rate of return of 8% per annum for ten
years, with an option to opt for pension on a monthly / quarterly / half yearly and annual basis. It provides
senior citizens more avenues to earn a steady income at a time when fixed deposit rates are low.The
differential return, i.e. the difference between the return generated by LIC and the assured return of 8%
per annum would be borne by Government of India as subsidy on an annual basis.
o Statement 3 is correct: According to the recent amendments by the government, the investment
limit of Rs 7.5 lakh per family in the existing scheme is enhanced to Rs 15 lakh per senior citizen in
the modified PMVVY, thereby providing a larger social security cover to the Senior citizens. It will
enable upto Rs.10000 Pension per month for Senior Citizens.This government sponsored socially-oriented
insurance scheme is available online as well as offline. Also, this plan is exempt from goods and service
tax.

Q 17.D
o India's oldest think-tank, the United Service Institution (USI) is helping to promote the niche concept
of 'Battlefield Tourism' to popularise historical sites where Indian soldiers have fought and died
over the ages. While battlefield tourism is in its infancy in India, there are parts of the world where it is
the mainstay of the local economy.The USI is producing 'Battlefield Guides' focused on the battles of the
Indian army to enable aficionados to visit many exotic locations where they were fought.
o The first such tour is being conducted in May by the Yatra travel agency. Titled 'WW-I-India on the
Western Front: Journey from an Indian Soldier's Perspective', it will take visitors to various locations in
France and Flanders, where the Indian Corps held the line against the formidable German army.
o Rajasthan might soon add ‗battlefield tourism‘ into its portfolio to boost tourism in the region. This might
be a golden opportunity for travellers to relive the battles of past as if they are a part of history. Lieutenant
General Cherish Mathson, general officer commanding-in-chief of South Western Command of Indian
Army, will be introducing this concept to the hoteliers, which is already popular in the United States and
South Africa.
6 www.visionias.in ©Vision IAS

https://T.me/Civilsbuzz https://T.me/PDF4Exams
Downloaded From:- https://T.me/TestSeries4Exams

Q 18.A
o Statement 1 is correct: Blockchain Technology: It is a digital public ledger that records every
transaction. Once a transaction is entered in the blockchain, it cannot be erased or modified. Each
transaction is recorded to the ledger after verification by the network participants, mainly a chain of
computers, called nodes. Thus, a blockchain is a chain of blocks that contains data. Blockchain systems
are now comprised of two major components.
 The first component is peer to peer network. This is the mechanism by which the many computers
manage the database and communicate new changes to that database which are called transactions.
 Second major component of blockchain systems are the database itself and this database stores the
complete history of transaction and the order in which those transaction occurs.

o Statement 2 is not correct: Bitcoin or cryptocurrencies is one of the popular applications for the
technology. But there are multiple uses of blockchain beyond cryptocurrency - it has the power to
transform business processes and applications across sectors — from financial services to agriculture,
from healthcare to education, among others. Some examples include:
 Blockchain-powered smart contracts where every piece of information is recorded in a traceable
https://T.me/PDF4Exams

and irreversible manner would enhance ease of doing business, augment the credibility, accuracy and
efficiency of a contract and reduce the risk of frauds substantially.
 Property deals which are still carried out on paper making them prone to disputes, can be benefitted
through in-built transparency, traceability and efficiency in this system
 Financial services: For example, Yes Bank adopted this technology to fully digitise vendor financing
for one of its clients which enables timely processing of vendor payments without physical documents
and manual intervention while tracking the status of transactions in real time. Even NITI Aayog is
reportedly building a platform called 'IndiaChain' — a shared, India-specific blockchain infrastructure
to leverage the trinity of Jan-Dhan-Yojana, Aadhaar and the mobile.
 Healthcare and pharmaceuticals: It involves a lot of sensitive clinical data which demands a secure
and reliable system. • Insurance sector: It may play a crucial part in health or agriculture insurance
claims management by reducing the risk of insurance claim frauds. • Education sector to ensure time-
stamped repository of pass-outs and job records of students to enable easier verification of candidates
by the employees.

Q 19.C
o Natural justice is the concept of common law which implies fairness, reasonableness, equality and
equity. The principle of Audi Alteram Partemis the basic concept of the principle of natural justice.
This doctrine states the no one shall be condemned unheard. This ensures a fair hearing and fair
justice to both the parties. Under this doctrine, both the parties have the right to speak. No decision can
be declared without hearing both the parties. The aim of this principle is to give an opportunity to both the
parties to defend themselves. Hence, statement 1 is correct.
o In the Constitution of India, nowhere the expression 'Natural Justice' is used. According to the
Supreme court they are not incorporated but inherent principles of the Constitution. Hence, statement 2 is
correct.
o In India, the principles of natural justice are reflected in many constitutional provisions like Article 14, 15,
17, 19, 21 etc. Article 22 of the Indian Constitution guarantees natural justice and provides fair hearing to
the arrested persons. On violation of this Article a aggrieved person can seek constitutional remedies
under Article 32 and 226 from Supreme Court and High Court respectively. Hence, statement 3 is
correct.

7 www.visionias.in ©Vision IAS

https://T.me/Civilsbuzz https://T.me/PDF4Exams
Downloaded From:- https://T.me/TestSeries4Exams

Q 20.C
o The Union Cabinet, chaired by the Prime Minister Shri Narendra Modi has approved National Policy on
Biofuels – 2018.
o Salient Features -
 The Policy categorises biofuels as
 "Basic Fuels" - First Generation (1G) bioethanol ( ethanol produced from biomass such as
sugar containing materials, like sugar cane, sugar beet, sweet sorghum etc.; starch containing
materials such as corn, cassava, rotten potatoes, algae etc.; and, cellulosic materials such as
bagasse, wood waste, agricultural and forestry residues or other renewable resources like
industrial waste) & biodiesel (a methyl or ethyl ester of fatty acids produced from non-edible
vegetable oils, acid oil, used cooking oil or animal fat and bio-oil)
 "Advanced Biofuels" i.e . Fuels such as Second Generation (2G) Ethanol, Drop-in fuels, algae
based 3G biofuels, bio-CNG, bio-methanol, Di Methyl Ether (DME) derived from bio-methanol,
biohydrogen, drop in fuels with MSW as the source / feedstock material . These fuels which are
 produced from lignocellulosic feedstocks (i.e. agricultural and forestry residues, e.g. rice & wheat
straw/corn cobs & stover/bagasse, woody biomass), non-food crops (i.e. grasses, algae), or
https://T.me/PDF4Exams

industrial waste and residue streams


 having low CO2 emission or high GHG reduction and do not compete with food crops for land
use.
 The Policy expands the scope of raw material for ethanol production by allowing use of Sugarcane
Juice, Sugar containing materials like Sugar Beet, Sweet Sorghum, Starch containing materials like
Corn, Cassava, Damaged food grains like wheat, broken rice, Rotten Potatoes, unfit for human
consumption for ethanol production.
 Farmers are at a risk of not getting appropriate price for their produce during the surplus production
phase. Taking this into account, the Policy allows use of surplus food grains for production of ethanol
for blending with petrol with the approval of National Biofuel Coordination Committee.
 With a thrust on Advanced Biofuels, the Policy indicates a viability gap funding scheme for 2G
ethanol Bio refineries of Rs.5000 crore in 6 years in addition to additional tax incentives, higher
purchase price as compared to 1G biofuels.
 The Policy encourages setting up of supply chain mechanisms for biodiesel production from non-
edible oilseeds, Used Cooking Oil, short gestation crops.
 Roles and responsibilities of all the concerned Ministries/Departments with respect to biofuels has
been captured in the Policy document to synergise efforts.

Q 21.C
o The Constitution provides for categorization of fundamental rights as follows:
 Right to equality(Articles 14-18)
 Right to freedom(Articles19-22)
i. Protection of six rights regarding freedom of : speech and expression; assembly; associations;
movement;residence; and profession (Article 19)
ii. Protection in respect of conviction for offences (Article 20)
iii. Protection of life and personal liberty (Article 21). Hence statement 2 is not correct.
iv. Right to elementary education (Artile 21A)
v. Protection against arrest and detention in certain cases (Article 22)
 Right against exploitation (Articles 23 and 24)
i. Prohibition of traffic in human beings and forced labour(Article 23)
ii. Prohibition of employment of children in factories, etc.(Articled 24)
 Right to freedom of religion (Article 25-28)
8 www.visionias.in ©Vision IAS

https://T.me/Civilsbuzz https://T.me/PDF4Exams
Downloaded From:- https://T.me/TestSeries4Exams

i. Freedom of conscience and free profession, practice and propagation of religion (Article 25).
Hence statement 3 is correct.
ii. Freedom to manage religious affairs. (Article 26)
iii. Freedom as to payment of taxes for promotion of any particular religion (Article 27)
iv. Freedom as to attendance at religious instruction or religious worship in certain educational
institutions (Article 28)
 Cultural and educational rights(Article 29 and 30)
i. Protection of interests (culture and language) of minorities (Article 29). Hence statement 1 is
not correct.
ii. Right of minorities to establish and administer educational institutions (Article 30)

Q 22.D
o Statement 1 is not correct: the Constitution doesn‘t prescribe any measures or authority for enforcing
fundamental duties. It is incumbent on all organs of State i.e. legislature, executive, judiciary and society
at large to nurture and enforce fundamental duties.
o Statement 2 is not correct: All the provisions of fundamental duties are applicable to only citizens.
https://T.me/PDF4Exams

Q 23.C
o Any citizen of India of full age and capacity can make a declaration renouncing his Indian citizenship.
Upon the registration of that declaration, that person ceases to be a citizen of India. However, if such a
declaration is made during a war in which India is engaged, its registration shall be withheld by the
Central Government.
o Further, when a person renounces his Indian citizenship, every minor child of that person also loses
Indian citizenship. However, when such a child attains the age of eighteen, he may resume Indian
citizenship. Hence option (c) is correct.
o Also, India, as well as Canada, have a single citizenship provision. So a person can be acquired either
Indian citizenship or Canadian citizenship at a time, but not both simultaneously.

Q 24.B
o Article 38 (DPSP) in the Constitution Of India 1949 says: The State shall strive to promote the welfare of
the people by securing and protecting as effectively as it may a social order in which justice, social,
economic and political, shall inform all the institutions of the national life
o Preamble of our constitution reads as follows: WE, THE PEOPLE OF INDIA, have solemnly resolved to
constitute India into a SOVEREIGN SOCIALIST SECULAR DEMOCRATIC REPUBLIC and to secure
to all its citizens: JUSTICE, social, economic and political; LIBERTY of thought, expression, belief, faith
and worship; EQUALITY of status and of opportunity; and to promote among them all FRATERNITY
assuring the dignity of the individual and the unity and integrity of the Nation; IN OUR CONSTITUENT
ASSEMBLY this twenty-sixth day of November, 1949, do HEREBY ADOPT, ENACT AND GIVE TO
OURSELVES THIS CONSTITUTION.
o Even though the objective of fundamental rights is to ensure political justice, the word 'political justice'
does not find mention in this part.

Q 25.A
o Statement 1 is correct: South Asia Wildlife Enforcement Network (SAWEN) is an inter-governmental
wildlife law enforcement support body of South Asian countries namely- Afghanistan, Bangladesh,
Bhutan, India, Maldives, Nepal, Pakistan and Sri Lanka. It is aimed at combating wildlife crime in
South Asia. It focuses on policy harmonization; institutional capacity strengthening through knowledge
and intelligence sharing; and collaboration with regional and international partners to enhance wildlife
law enforcement in the member countries. Recently the first meeting of SAWEN was held in India to curb
wildlife crime in the South Asian region.
9 www.visionias.in ©Vision IAS

https://T.me/Civilsbuzz https://T.me/PDF4Exams
Downloaded From:- https://T.me/TestSeries4Exams

o Statement 2 is not correct: SAWEN has not been established under the aegis of South Asian
Association for Regional Cooperation (SAARC). It was formally launched during the Second
Meeting of the South Asia Experts Group on Illegal Wildlife Trade held in 2011 in Paro,
Bhutan. The meeting agreed to an action-oriented work plan, establishment of SAWEN Secretariat to be
hosted by the Government of Nepal, governance and operational structure of SAWEN, and the need for
strategic collaboration on communications and fund-raising.
o Statement 3 is not correct: SAWEN is now a legitimate intergovernmental organization with
endorsement of the SAWEN Statute by five countries namely Sri Lanka, India, Nepal, Pakistan and
Bangladesh. India adopted the statute in 2016.

Q 26.C
o Kaveri river is often seen in the news due to inter-state water dispute, which is increasing because
the river is dying in the recent past. Earlier the Kaveri continued to flow throughout the year but
today river has water only three-four months in a year. In this background, Supreme Court
ordered the constitution of a Cauvery Management Board.
o Noyyal is a major tributary of Cauvery river. It rises from the Vellingiri hills in the Western Ghats in
Tamil Nadu.
https://T.me/PDF4Exams

o Krishnaraja Sagara Dam was built on the river Cauvery.


o Kaveripoompattinam or Poompuhar, in Nagapattinam district of Tamil Nadu, where the Cauvery meets
the sea, was once a busy port city and the capital of the Chola kings. Through this port maritime trade
prospered during the Chola period. Hence, option (c) is correct.

Q 27.A
o Statement 1 is correct: The Industrial and Commercial Bank of China (ICBC), a top state-run Chinese
bank has launched China‘s first India-dedicated publicly offered investment fund, named the Industrial
and Commercial Bank of China (ICBC) Credit Suisse India Market Fund.
o Statement 2 is not correct: It will invest in exchange-traded funds listed on more than 20 exchanges in
Europe and the U.S. that are based on the Indian market.

Q 28.C
o Due to higher sugar production against the estimated consumption during the current sugar season 2017-
18, the domestic sugar prices have remained depressed since the beginning of the season.
o Due to depressed market sentiments and crash in sugar prices, the liquidity position of sugar mills has
been adversely affected, leading to accumulation of cane price dues of farmers which have risen to more
than Rs.19,000 crore.
o In a bid to ease financial difficulties faced by sugarcane farmers on account of mounting arrears from
sugar mills, The Cabinet Committee of Economic Affairs (CCEA) approved a production-linked
subsidy of Rs 55 per tonne (Rs 5.5 per quintal) for cane farmers.

Q 29.D
o Democracy is a form of government in which all eligible citizens have an equal say in the decisions that
affect their lives. Democracy allows people to participate equally—either directly or through elected
representatives—in the proposal, development, and creation of laws.
o A non-democratic form of government is where an individual or a single-party concentrates all power.
Some non-democratic governments can be classified into categories such as:
o Monarchy is a form of government in a state is ruled by an individual who typically inherits the throne by
birth and rules for life or until abdication.
o An oligarchy is a form of power structure in which power effectively rests with a small number of people.
These people could be distinguished by royalty, wealth, family ties, education, corporate, or military
control.
10 www.visionias.in ©Vision IAS

https://T.me/Civilsbuzz https://T.me/PDF4Exams
Downloaded From:- https://T.me/TestSeries4Exams

o Technocracy is a form of government in which experts in technology would be in control of all decision
making. Scientists, engineers, and technologists who have knowledge, expertise, or skills, would compose
the governing body, instead of politicians, businessmen, and economists.
o A theocracy is a form of government in which religious leaders acting in the place of God rule the state.
It is a form of government in which official policy is governed by immediate divine guidance or by
officials who are regarded as divinely guided or is pursuant to the doctrine of a particular religion or
religious group.
o An aristocracy is a form of government in which a few elite citizens rule; this is usually contrasted with
democracy, in which all citizens are able to rule.
o Hence all the above options are correct.

Q 30.B
o Statement 1 is not correct: Swaran Singh committee was constituted in 1976 to study constitution and
suggest amendments. It was also referred to as Committee on constitutional amendments. It formed the
basis for Part IVA i.e. Fundamental duties of Indian Constitution.The committee recommended an eight-
point code of fundamental duties. While, a number of these recommendations were accepted, some of
them were not, like -:
 To respect every democratic institutions enshrined in the Constitution and to avoid doing anything
https://T.me/PDF4Exams

that may impair their dignity and authority.


 To abjure communalism in any form or manner.
 To pay taxes as is required by laws of the land.
o Statement 2 is correct: The committee recommended that no law imposing such penalty or punishment
for implementing fundamental duties shall be called in question in any court or on the ground of
infringement of any of fundamental rights or on the ground of repugnancy to any other provision of the
constitution. Hence, it provided for implementation of fundamental duties to be kept out of judicial
review.

Q 31.A
Features of the Government of India Act of 1858:

o It provided that India henceforth was to be governed by, and in the name of, Her Majesty. It
changed the designation of the Governor-General of India to that of Viceroy of India. He (viceroy)
was the direct representative of the British Crown in India. Lord Canning thus became the first Viceroy of
India. Hence only statement 3 is not correct.
o It ended the system of double government by abolishing the Board of Control and Court of
Directors.
o It created a new office, Secretary of State for India, vested with complete authority and control over
Indian administration. The secretary of state was a member of the British cabinet and was responsible
ultimately to the British Parliament.
o It established a 15-member Council of India to assist the secretary of state for India. The council was an
advisory body. The secretary of state was made the chairman of the council.
o It constituted the secretary of state-in-council as a body corporate, capable of suing and being sued in
India and in England.

Q 32.C
o The term ‗liberty‘ means the absence of restraints on the activities of individuals, and at the same time,
providing opportunities for the development of individual personalities.
o The Preamble secures to all citizens of India liberty of thought, expression, belief, faith and worship,
through their Fundamental Rights, enforceable in the court of law, in case of violation. Hence, statement
1 is correct.
o Liberty, as elaborated in the Preamble, is very essential for the successful functioning of the Indian
democratic system. However, liberty does not mean ‗license‘ to do what one likes and has to be enjoyed
11 www.visionias.in ©Vision IAS

https://T.me/Civilsbuzz https://T.me/PDF4Exams
Downloaded From:- https://T.me/TestSeries4Exams

within the limitations mentioned in the Constitution itself. In brief, the liberty conceived by the
Preamble or fundamental rights is not absolute but qualified. Hence statement 3 is not correct.
o The ideals of liberty, equality and fraternity in our Preamble have been taken from the French
Revolution (1789–1799), while the ideal of justice—social, economic and political—has been taken from
the Russian Revolution (1917). Hence statement 2 is not correct.
Q 33.B
o It was in 1934 that the idea of a Constituent Assembly for India was put forward for the first time by
Manabendra Nath Roy, a pioneer of the communist movement in India. In 1935, the Indian National
Congress (INC), for the first time, officially demanded a Constituent Assembly to frame the Constitution
of India. Hence, statement 1 is correct.
o The Constituent Assembly was constituted in November 1946 under the scheme formulated by the
Cabinet Mission Plan. Hence, statement 2 is not correct.
o Seats allocated to each British province were to be divided among the three principal communities—
Muslims, Sikhs and general (all except Muslims and Sikhs), in proportion to their population.
o The representatives of each community were to be elected by members of that community in the
provincial legislative assembly and voting was to be by the method of proportional representation by
means of a single transferable vote. Hence, representatives of British provinces were indirectly
elected. Hence, statement 3 is correct.
https://T.me/PDF4Exams

Q 34.C
o A Presidential System is a form of government in which the president is the chief executive and is elected
directly by the people. In this system, all three branches – executive, legislative, and judiciary – are
constitutionally independent of each other, and no branch can dismiss or dissolve any other.
o Features of the presidential system:
 Head of the state is popularly elected. Hence statement 1 is correct.
 The head of state is also the head of the government (cabinet). Hence statement 2 is not correct.
 The executive is constitutionally independent of the legislature in respect to the duration of his or their
tenure and responsible to it for his or their political policies. Hence statement 3 is not correct.
 During his tenure parliament can neither appoint nor remove the government except according to the
impeachment process.The president is not a part of the legislature. The president is also not merely
the titular executive but he is the real executive and actually exercises the power, which the
constitution and the law confer upon him. Executive power is thus vested in an independently elected
president who is not directly accountable to or removable by the parliament. This type is found in the
USA and in many Latin American countries.

Q 35.A
o Fundamental duties are contained in article 51A of Indian constitution. They were inserted by 42nd
Constitution amendment on the recommendations of Swaran Singh Committee.
o Statement 1 is correct: Article 51A(a) - To promote harmony and the spirit of common brotherhood
amongst all the people of India transcending religious, linguistic and regional diversities, to renounce
practices derogatory to the dignity of women;
o Statement 2 is correct: Article 51A(c) - To uphold and protect the sovereignty, unity and integrity of
India;
o Statements 3 and 4 are not correct: Raising voice against injustice and supporting bonafide government
initiatives are not a part of fundamental duties.
o Complete list of Fundamental Duties
o It shall be the duty of every citizen of India:
 to abide by the constitution and respect its ideal and institutions;
 to cherish and follow the noble ideals which inspired our national struggle for freedom;
 to uphold and protect the sovereignty, unity and integrity of India;
 to defend the country and render national service when called upon to do so;

12 www.visionias.in ©Vision IAS

https://T.me/Civilsbuzz https://T.me/PDF4Exams
Downloaded From:- https://T.me/TestSeries4Exams

 to promote harmony and the spirit of common brotherhood amongst all the people of India
transcending religious, linguistic and regional diversities, to renounce practices derogatory to the
dignity of women;
 to value and preserve the rich heritage of our composite culture;
 to protect and improve the natural environment including forests, lakes, rivers, and wild-life and to
have compassion for living creatures;
 to develop the scientific temper, humanism and the spirit of inquiry and reform;
 to safeguard public property and to abjure violence;
 to strive towards excellence in all spheres of individual and collective activity, so that the nation
constantly rises to higher levels of endeavor and achievement.
 to provide opportunities for education to his child or ward between the age of six and fourteen years.
(Added by the 86th Constitutional Amendment)

Q 36.D
o IT corridors have been established between India and China to give Indian businesses a gateway
into the neighbouring country‘s thriving software market. India has been demanding China to provide
market access to Indian IT and pharmaceutical firms for several years to reduce bilateral trade deficit. The
two corridors, which were started by NASSCOM in collaboration with China‘s provincial governments,
https://T.me/PDF4Exams

are expected to provide the much-needed big opening for Indian IT firms.
o Guiyang Corridor is the second such initiative between India and China. The Dalian Corridor
collaborative technology platform launched in December last year focused on internet-of-things
(IoT). The Giuyang partnership will be centred around big data.
o Hence correct answer is option (d).

Q 37.D
o Article 19: Right To Freedom
o Under Article 19(1) All Citizen shall have the right—
 to freedom of speech and expression;
 to assemble peaceably and without arms;
 to form associations or unions;
 to move freely throughout the territory of India;
 to reside and settle in any part of the territory of India; and
 to practice and profession, or to carry on any occupation, trade or business.

Q 38.D
o Following are points for comparison between the two federation types :
o Indian federation is not the result of an agreement between States but union of states where as American
Federation is the result of an agreement between States. Hence, option (a) is not the correct answer.
o The fundamental difference between Indian Government and U.S. Government is that India is a
Parliamentary form of Government and U.S. Government is Presidential form of Government. Hence,
option (b) is not the correct answer.
o There is single citizenship for both the States and Union in India where as there is dual citizenships - one
Federal Citizenship another State Citizenship in the American constitution. Hence, option(c) is not the
correct answer.
o The Supreme Court has been given very wide powers, including appellate (Civil and criminal)
jurisdiction. It is the apex court and interprets the constitution , the constitutional matters are purely
interpreted by the supreme court in both the systems. Hence, the correct answer is option (d).

Q 39.B
o Preamble can be altered by an amendment under Article 368.
o Any part of the Constitution may be amended as long as the basic structure of the Constitution is not
violated. Amendment under Article 368 means that to be passed, it requires a special majority (i.e.,
13 www.visionias.in ©Vision IAS

https://T.me/Civilsbuzz https://T.me/PDF4Exams
Downloaded From:- https://T.me/TestSeries4Exams

more than half of the membership and more than two-thirds of the members present and voting) in
both the Houses of Parliament (i.e., Lok Sabha and Rajya Sabha).
o The Preamble has only once been amended in 1976, by the 42nd Constitutional Amendment Act. The Act
added three new words: ―socialist‖, ―secular‖ and ―integrity‖ to the Preamble.

Q 40.B
o Article 22 grants protection to persons who are arrested or detained.
o Detention is of two types: Punitive & Preventive
o Punitive: It is to punish a person for an offence committed by him after trial and conviction in a court. Its
purpose is not to punish a person for a past offence.
o Preventive: It means detention of a person without trial and conviction by a court. Its purpose is to
prevent him from committing an offence in the near future. Thus, preventive detention is only a
precautionary measure.
o The Constitution has divided the legislative power with regard to preventive detention between the
Parliament and the state legislatures.
o The Parliament has exclusive authority to make a law of preventive detention for reasons connected
with - defence, foreign affairs and the security of India. Hence only option (2) is correct.
o Both the Parliament as well as the state legislatures can concurrently make a law of preventive
https://T.me/PDF4Exams

detention for reasons connected with the security of a state, the maintenance of public order and the
maintenance of supplies and services essential to the community.
o Option (1), (3) and (4) are not correct.
o Hence the option (b) is the correct answer.

Q 41. A
o Statement 1 is not correct: Fundamental duties are mostly a feature of socialist countries like erstwhile
USSR etc. Unlike democratic countries, which are marked by predominance of fundamental rights,
socialist countries give equal importance to the fundamental rights and duties of their citizens.
o Statement 2 is not correct: Japanese constitution was among the earliest democratic constitutions
(enacted on May 3, 1947) to have specific provisions for fundamental duties. The provision for
fundamental duties in India constitution were added by 42nd amendment, 1976.
o Statement 3 is correct: Constitutions of major democratic countries do not specifically contain a list of
duties of citizens. Some of the major countries included are USA, Canada, France, Germany, Australia
etc.

Q 42.A
o Constitutional monarchy is a system of government in which a monarch (king/queen) shares power with a
constitutionally organized government. Constitutional monarchy differs from absolute monarchy (in
which a monarch holds absolute power), in that constitutional monarchs are bound to exercise their
powers and authorities within the limits prescribed within an established legal framework. Hence
statement 1 is correct.
o While monarchy is unelected, unlike an elected presidency, constitutional monarchy allows for certain
powers of the monarch to be limited and balanced by an elected body in the form of a Parliament of
elected ministers. It is often seen that constitutional monarchy possesses two central positive features:
 While presidents may see themselves in terms of a limited term of office, with them often being
"retired" from other posts into the presidency, constitutional monarchy tends to involve a professional
life-long commitment.
 Monarchs often do not represent specific political views, and that they provide stability or act as a
symbol of the state or nation. Hence statement 2 is correct.
o Constitutional monarchies:
o In Europe: United Kingdom, Netherlands, Belgium, Norway, Denmark, Spain, Luxembourg, Monaco,
Liechtenstein, Sweden etc.

14 www.visionias.in ©Vision IAS

https://T.me/Civilsbuzz https://T.me/PDF4Exams
Downloaded From:- https://T.me/TestSeries4Exams

o In Asia: Japan, Thailand, Bhutan etc Hence Statement 3 is not correct.


o Hence option (a) is the correct answer.

Q 43.D
o The Supreme Court of India has held in Union of India v. Naveen Jindal and Anr. that hoist the National
Flag by citizens is a form of freedom of speech and expression and hence it comes under Article
19(1) (a), thus it is a Fundamental Right.
o Right to freedom of conscience is under Article 25 of the Indian Constitution is a Fundamental
Right. Right to freedom of conscience is the inner freedom of an individual to mould his religious belief
and faith and State can not impose any restriction on this.
o Right to sleep is a Fundamental Right contained in the Article 21 of the Indian Constitution. A
bench of Justices B S Chauhan and Swatanter Kumar was unanimous on their verdict in 2016 on the
police action on Baba Ramdev's rally that Sleep is essential for a human being to maintain the delicate
balance of health necessary for its very existence and survival. Sleep is, therefore, a fundamental and
basic requirement without which the existence of life itself would be in peril under Article 21 of the
Constitution.
o Hence option (d) is correct.
https://T.me/PDF4Exams

Q 44.C
o The phrase ‗Directive Principles of State Policy‘ denotes the ideals that the State should keep in mind
while formulating policies and enacting laws. These are the constitutional instructions or
recommendations to the State in legislative, executive and administrative matters.
o Although these directives are non-justiciable, they are fundamental to the governance of the country. Dr.
B R Ambedkar had pointed out that the Directives have great value because they lay down that the goal of
Indian polity is ‗economic democracy‘ as distinguished from ‗political democracy‘.Sir B N Rau, the
constitutional advisor to the Constituent Assembly, stated that the Directive Principles are intended as
‗moral precepts for the authorities of the state. They have at least an educative value.‘ The Constitution
does not aim at preparing the masses for a communist struggle. While some directive principles draw on
Indian traditions (eg: organization of village panchayats), as a category they do not rely on ancient Indian
wisdom to solve the problems of the modern world (eg: organization of agriculture and animal husbandry
on modern and scientific lines).

Q 45.C
o Statement 1 is not correct: Community reserves are protected areas categories which were first
introduced in the Wildlife (Protection) Amendment Act of 2003 (the amendment to the Wildlife
Protection Act of 1972). These categories were added because of reduced protection in and around
existing or proposed protected areas due to private ownership of land, and land use.
o Statement 2 is correct: Community reserves can be declared by the State Government in any private
or community land, not comprised within a National Park, Sanctuary or a Conservation Reserve, where
an individual or a community has volunteered to conserve wildlife and its habitat. Community
Reserves are declared for the purpose of protecting fauna, flora and traditional or cultural conservation
values and practices. As in the case of a Conservation Reserve, the rights of people living inside a
Community Reserve are not affected.
o Statement 3 is correct: Singchung Bugun Village Community Reserve has been awarded the Indian
Biodiversity Award, 2018 under the ―Conservation of Wildlife Species‖ category for conservation of
the Bugun Liocichla bird. It‘s a 17 km2 hotspot for biodiversity launched by Bugun community of
Singchung Village by joining hands with the Forest Department.
o Singchung Bugun Village Community Reserve is adjacent to the Eaglenest Wildlife Sanctuary in the West
Kameng district, Arunachal Pradesh. The Sanctuary is rich in both flora and fauna, with 545 species of
birds and various species of plants and animals, the sanctuary is spread over about 217 sq km. The
15 www.visionias.in ©Vision IAS

https://T.me/Civilsbuzz https://T.me/PDF4Exams
Downloaded From:- https://T.me/TestSeries4Exams

sanctuary is also known for the existence of Bugun Liocichla, a bird species presently found only in this
park.
Q 46.A
o PCI is a statutory body with the mandate to act as the watchdog to oversee the conduct of the print
media. It derives its mandate from Press Council Act, 1978.
o It consists of Chairman (who has by convention, been a retired judge of Supreme Court) and 28
other members off whom 20 represent press, five are nominated from two Houses of Parliament and three
represent cultural, literary and legal fields. Hence statements 1 and 2 are correct.
o Chairman is selected by a committee headed by the Chairman of the Rajya Sabha. Other members are
the Speaker of Lok Sabha and a person elected by the members of the Council from among themselves.
o The term of the Chairman and the members of the Council is three years. A retiring member is eligible for
renomination for not more than one term.
o It is a quasi-judicial body which acts as a watchdog of the press. It adjudicates the complaints against and
by the press for violation of ethics and for violation of the freedom of the press respectively. However, It
does not have any penal powers to punish delinquent newspapers & journalists. Hence statement 3 is not
correct.
o Regarding the penal powers, suggestions have been made to the Council that it should have penal powers
https://T.me/PDF4Exams

to punish the delinquent newspapers/journalists. In response, the Council has consistently taken the view
that the moral sanctions provided to it under the existing scheme of the Act are adequate. It has opined
that in the prevalent conditions penal powers could tend to be misused by the authorities to curb the
freedom of the Press.
o Recently former Supreme Court judge C K Prasad has been appointed the chairman of the Press Council
of India (PCI) for a second term.
o Prasad headed the media watchdog for three years from November 2014. He had succeeded Justice
Markanday Katju.
o Hence the option (a) is the correct answer.

Q 47.D
o A Constitution performs the following basic functions:
 To provide a set of basic rules that allow for minimal coordination amongst members of a
society.
 To specify who has the power to make decisions in a society. It decides how the government will
be constituted.
 To set some limits on what a government can impose on its citizens. These limits are fundamental
in the sense that the government may never trespass them.
 To enable the government to fulfill the aspirations of a society and create conditions for a just
society.
o Hence, all the statements are correct.

Q 48.C
o The concept of equality implies that all people, as human beings, are entitled to the same rights and
opportunities to develop their skills and talents, and to pursue their goals and ambitions. Hence
statement (c) is correct.
o The idea of equality does not imply the elimination of all forms of differences. It merely suggests that
the treatment we receive and the opportunities we enjoy must not be pre-determined by birth or social
circumstance. Hence statement (b) is not correct.
o Political equality by itself is not sufficient to build egalitarian and just society. But it is certainly an
important component of it. Hence statement (a) is not correct.
16 www.visionias.in ©Vision IAS

https://T.me/Civilsbuzz https://T.me/PDF4Exams
Downloaded From:- https://T.me/TestSeries4Exams

Q 49.B
o Indian Independence Act was passed in 1947.
o It ended the British rule in India and declared India as an independent and sovereign state from August
15, 1947.
o It provided for the partition of India and creation of two independent dominions of India and Pakistan
with the right to secede from the British Commonwealth.The Constitution as adopted on November 26,
1949, contained a Preamble, 395 Articles and 8 Schedules.Some provisions of the Constitution pertaining
to citizenship, elections, provisional parliament, temporary and transitional provisions, and short title
contained in Articles 5, 6, 7, 8, 9, 60, 324, 366, 367, 379, 380, 388, 391, 392 and 393 came into force on
November 26, 1949 itself.
o India became a federal, democratic republic after its constitution came into effect on 26 January
1950. Hence statement 1 is not correct.The remaining provisions (the major part) of the Constitution
came into force on January 26, 1950. This day is referred to in the Constitution as the ‗date of its
commencement‘, and celebrated as the Republic Day.
o Republic and the ideals of liberty, equality and fraternity in the Preamble are borrowed from
French Constitution. Hence statement 2 is correct.In a republic, the head of the state is always elected
directly or indirectly for a fixed period. In India President is head of the state and is indirectly elected
while in the USA, he is directly elected. The term ‗republic‘ in our Preamble indicates that India has an
elected head called the president. He is elected indirectly for a fixed period of five years. Hence
https://T.me/PDF4Exams

statement 3 is not correct.


o Hence option (b) is the correct answer.

Q 50.D
o Statements 1 and 3 are not correct: The biennial Rim of the Pacific (RIMPAC) exercise is the largest
international maritime exercise in the world, hosted by the US, and in 2018, it was conducted with 26
countries including India. The Indian Navy deployed INS Sahyadri, an indigenously built Shivalik Class
stealth multirole frigate, for the exercise in 2018.
o RIMPAC 2018 was the 26th exercise in the series that began in 1971.
o Statement 2 is not correct: The exercise was held from June 27 to August 2, 2018, in and around the
Hawaiian Islands and Southern California.The capabilities demonstrated ranged from disaster relief and
maritime security operations to sea control and complex warfighting. The theme of RIMPAC 2018 was
"Capable, Adaptive, Partners." In 2018, though China was initially invited and had been participating
since 2014, the US withdrew that invitation after China‘s militarisation of disputed features in the South
China Sea.

Q 51.A
o Statement 1 is correct: Privacy is an integral component to have a life with liberty and dignity. It is on
this principle, that Right to privacy has been declared as a fundamental right under article 21.
o Statement 2 is not correct:. In addition, article 21 can‘t be suspended during the imposition of national
emergency.
o The SC in the case of KS Puttaswamy and others vs Union of India and others that 'Right to Privacy' is an
integral part of Right to Life and Personal Liberty guaranteed in Article 21 of the Constitution. It added
that the right to privacy is intrinsic to the entire fundamental rights chapter of the Constitution.

Q 52.C
o Statement 1 is correct and statement 2 is not correct: An Indian citizen who is ordinarily residing
outside India and holds an Indian Passport is considered as Non-Resident Indian. Since he carries an
Indian passport so he does not require visa whenever he intends to travel to India.
o Statement 3 is not correct: An NRI can have voting rights. According to the provisions of the
Representation of the People (Amendment) Act, 2010, a person who is a citizen of India and who has not
acquired the citizenship of any other country and is otherwise eligible to be registered as a voter and who
is absenting from his place of ordinary residence in India owing to his employment, education or
otherwise is eligible to be registered as a voter in the constituency in which his place of residence in India
as mentioned in his passport is located.

17 www.visionias.in ©Vision IAS

https://T.me/Civilsbuzz https://T.me/PDF4Exams
Downloaded From:- https://T.me/TestSeries4Exams

Q 53.C
o Rule of law refers to the supremacy of law: that society is governed by law and this law applies
equally to all persons, including government and state officials.
o Following basic principles of constitutionalism, common institutional provisions used to maintain the rule
of law include the separation of powers, judicial review, the prohibition of retroactive legislation and
habeas corpus. Genuine constitutionalism, therefore, provides a minimal guarantee of the justice of both
the content and the form of law.
o On the other hand, constitutionalism is safeguarded by the rule of law. Only when the supremacy of
the rule of law is established, can supremacy of the Constitution exist. Constitutionalism additionally
requires effective laws and their enforcement to provide structure to its framework.
o Hence, both the statements are correct.

Q 54.D
o 86th Amendment:
o Insertion of new Article 21A- After article 21 of the Constitution, the following article shall be inserted,
namely:
o Right to education: "21A. The State shall provide free and compulsory education to all children of the
https://T.me/PDF4Exams

age of six to fourteen years in such manner as the State may, by law, determine".
o Substitution of new article for article 45. - For article 45 of the Constitution, the following article shall be
substituted, namely: Provision for early childhood care and education to children below the age of six
years. "45. The State shall endeavour to provide early childhood care and education for all children until
they complete the age of six years".
o Amendment of article 51A. In article 51A of the Constitution, after clause (J), the following clause shall
be added, namely: "(k) who is a parent or guardian to provide opportunities for education to his child or,
as the case may be, ward between the age of six and fourteen years".
o Thus it has bearing on part III, IV and IVA of the constitution.

Q 55.D
o Various features of the constitution borrowed from different sources are as follows :
 A Government of India Act, 1935: Federal Scheme, Office of Governor, Judiciary, Public Service
Commissions, Emergency provisions, Administrative details etc.
 British Constitution: Parliamentary government, Rule of Law, Legislative procedure, Single
citizenship, Cabinet system, Prerogative writs, Parliamentary privileges, Bicameralism.
 US Constitution: Fundamental Rights, Independence of Judiciary,Judicial Review, Impeachment of
the President,Post of Vice-President, Removal of Supreme Court and High Court judges. Hence, pair
2 is correctly matched.
 Irish Constitution: Directive Principles of State Policy, Method of election of President, Nomination
of members to Rajya Sabha. Hence, pair 4 is correctly matched.
 Canadian Constitution: Federation with a strong centre, Residuary powers with the centre,
Appointment of state governors by the centre, Advisory jurisdiction of the Supreme Court. Hence,
pair 1 is correctly matched.
 Australian Constitution: Concurrent List, Freedom of trade,Commerce and intercourse,Joint sitting
of the two Houses of Parliament. Hence, pair 3 is correctly matched.
 Weimar Constitution: Suspension of Fundamental Rights during Emergency.
 Soviet Constitution: Fundamental duties, The ideal of justice (social, economic and political) in the
Preamble.
 French Constitution: Republic ,The ideals of liberty, equality and fraternity in the Preamble
 South African Constitution: Procedure for amendment of the Constitution, Election of the members
of Rajya Sabha.
 Japanese Constitution: Procedure established by law.
18 www.visionias.in ©Vision IAS

https://T.me/Civilsbuzz https://T.me/PDF4Exams
Downloaded From:- https://T.me/TestSeries4Exams

Q 56.B
o PM recently inaugurated the state run NHPC Ltd‘s Kishanganga hydro power project in Jammu and
Kashmir.
o Statement 1 is not correct: It is a 330 megawatt Run of the River Hydroelectric power project located in
Gurez valley of Bandipora district in north Kashmir.
o Statement 2 is correct: It envisages diversion of water from the Kishanganga River to a power plant in
the Jhelum River basin through an underground tunnel and the discharge of the water into the Wular lake.
o Other Related Information:
 The project began in 2009 but in 2010 Pakistan appealed to Hague‘s Permanent Court of Arbitration
complaining that the project violated the Indus River Treaty and deprived Pakistan the water share to
its power project which is under construction at Neelam valley in PoK, as the Kishanganga river flows
into Pakistan.
 Pakistan is constructing its own 1,000 megawatts Neelum-Jhelum hydropower project with the
assistance of China on its side of the river.
 Court of Arbitration ordered India to submit technical data of the project and allowed India to go
ahead with the construction of the dam while maintaining minimum 9 cubic metres of flow of water
https://T.me/PDF4Exams

across border.
 12 per cent of the power generated from Kishanganga project will be given to the J&K as ―royalty‖,
apart from an extra 1 per cent for ―local development‖, while the rest will go into the national grid.
o Significance of Kishanganga Hydroelectric Project
 It is expected to give boost to the development of the region.
 It is an assertion by India over the territory of J&K and over its resources.
 The project has a great strategic value due to India‘s assertion of its rights under the Indus Waters
Treaty.

Q 57.A
o The Network for Spectrum(NFS) project, which is being implemented by state-run Bharat Sanchar
Nigam Limited (BSNL), is for laying of alternative communication network for defence services.
o The alternative communications network is being set up in lieu of airwaves vacated by the Defence
Ministry for civilian use.
o The defence ministry and Department of Telecommunications had signed a pact in 2009-10 under which
the former had agreed to vacate 25MHz of 3G spectrum and 20MHz in the 2G band in phases. In return,
DoT had committed to set up an exclusive defence network for its communication services.
o The NFS project will boost the communication capabilities of the Defence Forces in a major way leading
to enhanced national operational preparedness.
o The project will also have forward linkages to other related industries such as telecom equipment
manufacturing and other telecommunication related services.

Q 58.A
o The provisions in the Indian Constitution which deals with women rights and their welfare are as follows:-
 Article 14 expresses that the State shall not deny to any person the equality before the law and
equal protection of laws with in the territory of India.
 Article 15(1) prohibits the State to discriminate against any citizen on the grounds only of religion,
race, caste, sex, place of birth pr any of them.
 Article 15(3) permits the State to make special provisions for women and children.
 Article 16 provides that there shall be equality of opportunity for all citizens and they shall not
be discriminated on the basis of religion, race, caste and sex.
19 www.visionias.in ©Vision IAS

https://T.me/Civilsbuzz https://T.me/PDF4Exams
Downloaded From:- https://T.me/TestSeries4Exams

 Article 39(a) of the Constitution provides that the state in particular direct its policy towards
securing that citizen, men and women equally, have the right to an adequate means of
livelihood.
 Article 39(e) of the Constitution provides that the health and strength of workers, men and women,
and the tender age of children are not abused and that citizens are not forced by economic necessity to
enter avocations unsuited to their age or strength.
 Article 51(A)(e) of the Constitution provides that it will be the duty of every citizen to renounce
practices derogatory to the dignity of women.

Q 59.C
o Directive principles of state policy (DPSPs) are non-justiciable in nature as per Article 37 of Indian
constitution. Hence, they are not legally enforceable by the courts for their violation. Following were the
reasons for keeping them non-justiciable -
o Statement 1 is correct:DPSPs are aimed at establishing socio-economic democracy which requires huge
resources for their implementation. However, the country didn‘t possess sufficient financial resources
at the time of framing to implement them.
https://T.me/PDF4Exams

o Also, at the time of framing India was marked by the presence of vast diversity and backwardness.
Framers of the constitution held that these would stand in the way of implementation of DPSPs. It must be
noted that cultural diversity as such was not the reason which continues to be an integral component of
Indian society. It was the vast diversity w.r.t problems, priorities etc. and the backwardness of people w.r.t
education, health, skills etc. which stood in the way of implementation of DPSPs.
o Constitution framers thought that newly born independent Indian state with its many pre-occupations
might be crushed under the burden unless it was free to decide the order, the time, the place and the mode
of fulfilling them.
o Statement 2 is correct: Constitution framers also believed more in awakened public opinion rather
in-court procedures as the ultimate sanction for the fulfillment of these principles.

Q 60.D
o Article 32 gives a citizen the right to approach the Supreme Court to get any of the fundamental rights
restored in case of their violation. Under this article, the Supreme Court can issue orders and give
directives to the government for the enforcement of fundamental rights. The courts can issue various
special orders known as writs. Mandamus is a writ issued when the court finds that a particular
officeholder is not doing legal duty and thereby is infringing on the fundamental right of an individual.
o It cannot be issued to direct the central government to introduce an amendment bill in the Lok
Sabha as the government has no legal duty to do so. Debtors do not enjoy a fundamental right to have
their outstanding loans amicably resolved. So, mandamus under Article 32 cannot be issued. Similarly,
remission of sentence is not a fundamental right and Article 32 shall not apply here.
o Some other writs that are issued by the Courts are:
o Habeas corpus: A writ of habeas corpus means that the court orders that the arrested person should be
presented before it. It can also order to set free an arrested person if the manner or grounds of arrest are
not lawful or satisfactory.
o Prohibition: This writ is issued by a higher court (High Court or Supreme Court) when a lower court has
considered a case going beyond its jurisdiction.
o Quo Warranto: If the court finds that a person is holding office but is not entitled to hold that office, it
issues the writ of quo warranto and restricts that person from acting as an office holder.
o Certiorari: Under this writ, the court orders a lower court or another authority to transfer a matter pending
before it to the higher authority or court.
20 www.visionias.in ©Vision IAS

https://T.me/Civilsbuzz https://T.me/PDF4Exams
Downloaded From:- https://T.me/TestSeries4Exams

Q 61.A
o Samagra Shiksha is an integrated Scheme recently launched in May 2018 for school education extending
support to States from pre-school to senior secondary levels for the first time.
o The Scheme is a paradigm shift in the conceptual design of school education by treating ‗school‘
holistically as a continuum from pre-school, primary, upper primary, secondary and senior secondary
levels.
o It subsumes the three Schemes of Sarva Shiksha Abhiyan (SSA), Rashtriya Madhyamik Shiksha
Abhiyan (RMSA) and Teacher Education (TE). Hence option (a) is the correct answer.
o The major objectives of the scheme are provision of quality education and enhancing learning outcomes
of students; Bridging Social and Gender Gaps in School Education; Ensuring equity and inclusion at all
levels of school education; Ensuring minimum standards in schooling provisions; Promoting
Vocationalisation of education; Support States in implementation of Right of Children to Free and
Compulsory Education (RTE) Act, 2009; and Strengthening and up-gradation of SCERTs/State Institutes
of Education and DIET as a nodal agencies for teacher training.
o At the National level, it would be implemented by a Governing Council headed by Minister of Human
Resource Development and a Project Approval Board (PAB) headed by Secretary, Department of School
Education and Literacy. At the State/UT level the Scheme will be implemented as a Centrally Sponsored
Scheme by the Department through a single State Implementation Society (SIS).
https://T.me/PDF4Exams

o Rashtriya Uchchattar Shiksha Abhiyan is a holistic scheme of development for higher education in
India initiated in 2013 by the Ministry of Human Resource Development, Government of India. The
centrally sponsored scheme aims at providing strategic funding to higher education institutions
throughout the country.

Q 62.A
o Under Article 14 of the Constitution, the concept of ‗equality before law‘ is an element of the concept of
‗Rule of Law‘, propounded by A.V. Dicey, the British jurist.
o Equality before the law, that is, equal subjection of all citizens to the ordinary law of the land
administered by the ordinary law courts. The rule of equality before law is not absolute and there are
constitutional and other exceptions such as:
 The foreign sovereigns (rulers), ambassadors and diplomats enjoy immunity from criminal and
civil proceedings. Hence, statement 1 is correct.
 The UNO and its agencies enjoy the diplomatic immunity.
o Article 18 under Constitution of India abolishes titles and makes four provisions in that regard:
 It prohibits the state from conferring any title (except a military or academic distinction) on any body,
whether a citizen or a foreigner.
 It prohibits a citizen of India from accepting any title from any foreign state.
 A foreigner holding any office of profit or trust under the state cannot accept any title from any
foreign state without the consent of the president.
 No citizen or foreigner holding any office of profit or trust under the State is to accept any
present, emolument or office from or under any foreign State without the consent of the
President. Hence, statement 2 is not correct.

Q 63.C
o Recently, Union Cabinet has cleared a DNA Technology (Use and Application) Regulation Bill, 2018 that
allows law enforcement agencies to collect DNA samples, create ―DNA profiles‖ and special databanks
for forensic-criminal investigations.
o Andhra Pradesh has become India‘s first state that launch DNA profiling for criminals. Hence
statement 3 is correct.
o DNA can be collected from skin, hair, blood and even from saliva. Hence statement 1 is not correct.
o Following are the major advantages of DNA profiling:
o It helps in identifying missing children.
o It helps in determining parenting and solving complex criminal cases. Hence statement 2 is correct.
o In recognizing unclaimed dead bodies.
21 www.visionias.in ©Vision IAS

https://T.me/Civilsbuzz https://T.me/PDF4Exams
Downloaded From:- https://T.me/TestSeries4Exams

Q 64.D
o Statement 1 is not correct: While most of the fundamental rights are available against the state, a
number of them are also available against private citizens and private corporations -:
 Article 17 (against untouchability) and Article 23 (forced labor) are available against private
individuals.
 Article 15 (no discrimination against Religion, race, caste, sex, place of descent only, so economic
discrimination is present) and Article 24 (child labor) are available against private corporations
o Statement 2 is not correct: The constitution doesn‘t define the term ‗Untouchability‘
o Statement 3 is not correct: Untouchability refers to the social disabilities imposed on certain castes.
Thus, it doesn‘t cover the social boycott of a few individuals or their exclusion from religious service, etc.
Hence, the practice of social boycott of a family for not permitting a lower caste in its house will not
amount to practicing untouchability.

Q 65.C
o Article 21 (Protection Of Life And Personal Liberty) states that no person shall be deprived of his life or
personal liberty except according to procedure established by law. This article is available for both for
https://T.me/PDF4Exams

citizen and non-citizen.


o The object of the fundamental right under Article 21 is to prevent encroachment upon personal
liberty and deprivation of life except according to procedure established by law. It clearly means
that this fundamental right has been provided against state only.
o If an act of private individual amounts to encroachment upon the personal liberty or deprivation of life of
other person. Such violation would not fall under the parameters set for the Article 21 but in such a case
the remedy for aggrieved person would be either under Article 21. In such a case the remedy for
aggrieved person would be either under Article 226 of the constitution or under general law.
o Supreme Court through various judgments declared the following rights as part of Article 21:
 The right to go abroad.
 The right to privacy.
 The right against solitary confinement.
 The right against hand cuffing.
 The right against delayed execution.
 The right to shelter.
 The right against custodial death.
 The right against public hanging.
 Doctors assistance
Hence, both the statements are correct.

Q 66.B
o Originally, the Indian Constitution, like any other federal constitution, provided for a dual polity and
contained provisions with regard to the organisation and powers of the Centre and the states. Later, the
73rd and 74th Constitutional Amendment Acts (1992) have added a third-tier of government (i.e.,
local) which is not found in any other Constitution of the world. The 73rd Amendment Act of 1992
gave constitutional recognition to the panchayats (rural local governments) by adding a new Part IX17
and a new Schedule 11 to the Constitution. Similarly, the 74th Amendment Act of 1992 gave
constitutional recognition to the municipalities (urban local governments) by adding a new Part IX-A18
and a new Schedule 12 to theConstitution.

22 www.visionias.in ©Vision IAS

https://T.me/Civilsbuzz https://T.me/PDF4Exams
Downloaded From:- https://T.me/TestSeries4Exams

Q 67.B
o Following Committees were set up post-independence for state reorganisation:
 Dhar Committee: In June 1948, the Government of India appointed the Linguistic Provinces
Commission under the chairmanship of S K Dhar to examine the feasibility of this. The commission
submitted its report in December 1948 and recommended the reorganisation of states on the basis of
administrative convenience rather than a linguistic factor. Hence option 3 is correct.
 JVP Committee: Another Linguistic Provinces Committee by the Congress in December 1948 itself
to examine the whole question afresh. It consisted of Jawaharlal Nehru, Vallahbhai Patel and Pattabhi
Sitaramayya and hence, was popularly known as JVP Committee. It submitted its report in April 1949
and formally rejected language as the basis for reorganisation of states.
 Fazl Ali Commission: Government of India appointed (in December 1953) a three-member States
Reorganisation Commission under the chairmanship of Fazl Ali. Its other two members were K M
Panikkar and H N Kunzru. It submitted its report in September 1955 and broadly accepted language
as the basis of reorganisation of states. But, it rejected the theory of ‗one language– one state‘. Hence
option 1 is correct.
o Ashok Mehta Committee: In December 1977, the Janata Government appointed a committee
on panchayati raj institutions under the chairmanship of Ashok Mehta. Hence option 2 is not correct.

Q 68.C
https://T.me/PDF4Exams

o In the backdrop of the global financial crisis that started in 2007, the Basel Committee on Banking
Supervision (BCBS) proposed certain reforms to strengthen global capital and liquidity regulations. As
per global regulatory standards on liquidity, there are two minimum standards for funding
liquidity: Net Stable Funding Ratio or NSFR and LCR or Liquidity Coverage Ratio.
o Reserve Bank of India (RBI) released final guidelines prescribing 100 percent net stable funding
ratio (NSFR), a long-term liquidity measurement included in the Basel III liquidity standards, to
ensure banks maintain adequate liquid resources for more resilience. NSFR is defined as the amount of
available stable funding (ASF) relative to the amount of required stable funding (RSF). It promotes
resilience over a longer-term time horizon by requiring banks to fund their activities with more stable
sources of funding on an ongoing basis.
o RBI started phasing in implementation of the LCR from January 2015. LCR promotes short-term
resilience of banks to potential liquidity disruptions by ensuring that they have sufficient high quality
liquid assets (HQLAs) to survive an acute stress scenario lasting for 30 days.

Q 69.A
o A citizen has the right to approach a High Court or the Supreme Court to get any of the fundamental
rights restored in case of their violation. The Supreme Court and the High Courts can issue orders and
give directives to the government for the enforcement of rights.
o Caste Panchayats often decree or encourage ' honor killing' or other atrocities in an institutionalized way
on boys and girls of different castes and religions who wish to marry or are already married. This violates
the fundamental right to life and personal liberty of people. Hence, the High Courts may be
approached for the violation of this fundamental right.
o In the Constitution, originally, there was a fundamental right to ‗acquire, possess and maintain‘ property.
But the Constitution made it clear that property could be taken away by the government for public
welfare. In 1973, the Supreme Court gave a decision that the right to property was not part of the basic
structure of the Constitution and therefore, parliament had the power to abridge this right by an
amendment. In 1978, the 44th amendment to the Constitution removed the right to property from the
list of fundamental rights and converted it into a simple legal right under article 300 A.
o The fundamental right to freedom of religion is subject to certain limitations. The government can
impose restrictions on the practice of freedom of religion in order to protect public order, morality, and
health. This means that the freedom of religion is not an unlimited right. The government can interfere in
religious matters for rooting out certain social evils. For example in the past, the government has taken
steps banning practices like Sati, bigamy or human sacrifice. Such restrictions cannot be opposed in the
name of interference in the right to freedom of religion.

23 www.visionias.in ©Vision IAS

https://T.me/Civilsbuzz https://T.me/PDF4Exams
Downloaded From:- https://T.me/TestSeries4Exams

Q 70.A
o Statement 1 is correct: Article 30 grants the rights to the minorities to establish and administer
educational institutions. The protection under Article 30 is confined only to religious and linguistic
minorities and not to any sections of citizens.
o Article 30: Right of minorities to establish and administer educational institutions
 All minorities, whether based on religion or language, shall have the right to establish and administer
educational institutions of their choice(1A) In making any law providing for the compulsory
acquisition of any property of an educational institution established and administered by a minority,
referred to in clause ( 1 ), the State shall ensure that the amount fixed by or determined under such law
for the acquisition of such property is such as would not restrict or abrogate the right guaranteed under
that clause.
 The state shall not, in granting aid to educational institutions, discriminate against any educational
institution on the ground that it is under the management of a minority, whether based on religion or
language
o Statement 2 is not correct. According to Article 29 any section of the citizens residing in the territory of
India or any part thereof having a distinct language, script or culture of its own shall have the right to
https://T.me/PDF4Exams

conserve the same (29(1)). Further, no citizen shall be denied admission into any educational institution
maintained by the State or receiving aid out of State funds on grounds only of religion, race, caste,
language or any of them. (29(2)).
o The first provision protects the right of a group while the second provision guarantees the right of a citizen
as an individual irrespective of the community to which he belongs.
o Article 29 grants protection to both religious minorities as well as linguistic minorities. However, the
Supreme Court held that the scope of this article is not necessarily restricted to minorities only, as it is
commonly assumed to be. This is because of the use of words ‗section of citizens‘ in the Article that
include minorities as well as majority.

Q 71.A
o Fundamental Rights are not absolute , it means that they can be subjected to reasonable
restrictions. For example, right to absolute freedom of expression can lead to defamation,
incitement to an offense etc.
o Fundamental Rights are not 'sacrosanct', it means that they can be repealed or modified by amending the
Constitution.
o Fundamental Rights as the Political Rights of people means that there are limitations on the executive's
arbitrary power.

Q 72.C
o The 42nd Amendment Act of 1976 added four new Directive Principles to the original list.
o They require the State:
 To secure opportunities for healthy development of children (Article 39).
 To promote equal justice and to provide free legal aid to the poor (Article 39 A).
 To take steps to secure the participation of workers in the management of industries (Article 43 A).
 To protect and improve the environment and to safeguard forests and wild life (Article 48 A).
o The 44th Amendment Act of 1978 added one more Directive Principle which requires the State to
minimize inequalities in income, status, facilities and opportunities.
o Hence, the answer is (c).

24 www.visionias.in ©Vision IAS

https://T.me/Civilsbuzz https://T.me/PDF4Exams
Downloaded From:- https://T.me/TestSeries4Exams

Q 73.B
o Democracy is of two types—direct and indirect. In a direct democracy, the people exercise their
supreme power directly as is the case in Switzerland. There are four devices of direct democracy,
namely, Referendum, Initiative, Recall and Plebiscite. In an indirect democracy, on the other hand, the
representatives elected by the people exercise the supreme power and thus carry on the government and
make the laws. This type of democracy, also known as representative democracy, is of two kinds—
parliamentary and presidential.
o The devices of direct democracy are:
 Referendum is a procedure whereby a proposed legislation is referred to the electorate for settlement
by their direct votes.
 Initiative is a method by means of which the people can propose a bill to the legislature for
enactment.
 Recall is a method by means of which the voters can remove a representative or an officer before the
expiry of his term, when he fails to discharge his duties properly.
 Plebiscite is a method of obtaining the public opinion on any issue of public importance. It is
https://T.me/PDF4Exams

generally used to solve the territorial disputes.


o Public interest litigation means litigation for the protection of the public interest. It is litigation introduced
in a court of law, not by the aggrieved party but by the court itself or by any other private party. It is not
necessary, for the exercise of the court‘s jurisdiction, that the person who is the victim of the violation of
his or her right should personally approach the court. Public interest litigation is the power given to the
public by courts through judicial activism. PIL is not a device for either direct or indirect
democracy. Hence option 3 is not correct.

Q 74.B
o One Stop Centre (OSC) Scheme
o It is a Centrally Sponsored Sub-scheme under National Mission for Empowerment of women since 1st
April 2015.
o It is an initiative of Ministry of Women & Child Development.
o It is aimed at supporting women affected by violence in private and public spaces, within the family,
community and at the workplace. Hence option (2) is correct.
o Establishment of OSCs was one of the key component of Nirbhaya Fund. Every OSC is integrated with
newly operational Women's Helpline (181).
o Services Offered by OSC
o Emergency response and rescue services.
o Medical Assistance to women affected by Violence.
o Skilled counsellor to provide Socio-Psychological Support/ Counselling.
o Assistance to women in lodging FIR.
o Temporary shelter to aggrieved women.
o Legal Aid and Counselling.
o Video Conferencing to facilitate speedy and hassle-free police and court proceedings.
o Recently, Ministry of Women and Child Development has approved 100 additional One Stop Centres.
o Hence option (b) is the correct answer.

25 www.visionias.in ©Vision IAS

https://T.me/Civilsbuzz https://T.me/PDF4Exams
Downloaded From:- https://T.me/TestSeries4Exams

Q 75.D

Legislation Fundamental Right Directive Principles of State Policy


Protection of the Civil Rightso Article 17 – Abolition of o Article 46 - The State shall promote with
Act, 1976 Untouchability special care the educational and economic
interests of the weaker sections of the
people, and, in particular, of the Scheduled
Castes and the Scheduled Tribes, and shall
protect them from social injustice and all
forms of exploitation
Child Labour Prohibition ando Article 24 – Prohibition ofo Article 39 - State to strive to provide
Regulation Act, 1986 employment of Children in adequate means of livelihood, equal pay for
factories etc. equal work, resource distribution, safety of
citizens and healthy development of
Children.
National Food Security Act, o Article 21 – Right to life and
o Article 39(a) – State to ensure citizens‘
2013 personal liberty except right to an adequate means of livelihood
https://T.me/PDF4Exams

according to procedure o Article 47 - Duty of the State to raise the


established by law. Right to level of nutrition and the standard of
food is an important part living and to improve public health
of securing right to life.
o Also, right to health has been
declared by Supreme Court
within the purview of Article
21

Q 76.B
o The Second Schedule deals with the emoluments, allowances and privileges of only
 President
 Governor
 Comptroller and Auditor General (CAG)
 Supreme Court judges
 High Court judges
 Speaker and Deputy Speaker of Lok Sabha and state Legislative Assembly
 Chairman and Deputy Chairman of Rajya Sabha and State Legislative Council

Q 77.C
o Green Good Deeds, is a the societal movement to protect environment and promote good living in
the country.
o The movement was formally launched as a nation-wide campaign in January 2018. It enlisted the
cooperation of thousands of people – students, teachers, voluntary organisations, Residents Welfare
Associations and professionals to adopt these deeds. The Ministry of Environment, Forest & Climate
Change had drawn up a list of over 500 Green Good Deeds and asked people to alter their behaviour to
Green Good Behaviour to fulfil their Green Social Responsibility.
o Recently, the movement has has found acceptance by the global community. The BRICS Ministerial has
agreed to include ―Green Good Deeds‖ in its official agenda in the next Ministerial meeting.
o Hence, option (c) is the correct answer.

26 www.visionias.in ©Vision IAS

https://T.me/Civilsbuzz https://T.me/PDF4Exams
Downloaded From:- https://T.me/TestSeries4Exams

Q 78.C
o With respect to British provinces, the Government of India Act of 1935 had following provisions:
 It abolished dyarchy in the provinces and introduced ‗provincial autonomy‘ in its place. The provinces
were allowed to act as autonomous units of administration in their defined spheres. Dyarchy was
introduced at the center. Hence, option 1 is not correct.
 The Act introduced responsible governments in provinces, that is, the governor was required to act
with the advice of ministers responsible to the provincial legislature. This came into effect in 1937
and was discontinued in 1939. Hence, option 2 is correct.
 It introduced bicameralism in six out of eleven provinces. Thus, the legislatures of Bengal, Bombay,
Madras, Bihar, Assam and the United Provinces were made bicameral consisting of a legislative
council (upper house) and a legislative assembly (lower house). Hence, option 3 is correct.

Q 79.A
o The parliamentary system is also known as the ‗Westminster‘ model of government, responsible
government, and cabinet government. The Constitution establishes the parliamentary system not only at
the Centre but also in the states. The features of the parliamentary system in India are:
 Presence of nominal and real executives.
 Majority party rule.
https://T.me/PDF4Exams

 Collective responsibility of the executive to the legislature.


 Membership of the ministers in the legislature.
 The leadership of the prime minister or the chief minister.
 Dissolution of the lower House (Lok Sabha or Assembly).

Q 80.C
o Statement 1 is correct: Under the Constitution, a person shall be disqualified for being elected as a
member of Parliament if he is not a citizen of India or has voluntarily acquired the citizenship of a foreign
state or is under any acknowledgement of allegiance to a foreign state.
o Statement 2 is correct: Constitution confers the following rights and privileges on the citizens of India
which includes eligibility to hold certain public offices, that is, President of India, Vice-President of India,
judges of the Supreme Court and the high courts, governor of states, attorney general of India and
advocate general of states. In India both a citizen by birth as well as a naturalised citizen are eligible for
the office of President while in USA, only a citizen by birth and not a naturalised citizen is eligible for the
office of President.If a person has voluntarily acquired the citizenship of a foreign state, he ceases to have
citizenship of India and would be disqualified to run for office of President.

Q 81.C
o Recently, Indian Railways took the initiative to beautify stations with local art forms. The idea was to turn
railway stations into open-air, public-art museums so that there is greater awareness among people
regarding their destinations local art forms and what they have as the natural heritage of the country.
o For the first time, the Indian Railways‘ annual cleanliness contest included an award for beautification,
and the 62 entries from across 11 of the 17 railway zones were stunning examples of what is possible
when regional governments work with local artists.
o Madhubani in Bihar and Tamil Nadu‘s Madurai were tied in second place. The first station features
intricate Ramayana-themed work by over 140 artists. The latter is covered in vibrant temple art.
o Three stations — Gandhidham in Gujarat, Kota in Rajasthan and Secunderabad in Telangana —
won third place. The paintings at Gandhidham focused on the Kutch festival, Kota was covered in
Kota-Bundi art and Secunderabad showcased the state‘s Cheriyal form.
o Bundi School of Painting: Bundi School is an important school of the Rajasthani style of Indian miniature
painting that lasted from the 17th to the end of the 19th century in this princely state. The present district
of Bundi & Kota more or less corresponds to the medieval limits of Hada Chauhans where the art form
flourished.
27 www.visionias.in ©Vision IAS

https://T.me/Civilsbuzz https://T.me/PDF4Exams
Downloaded From:- https://T.me/TestSeries4Exams

o Madhubani: Madhubani painting is a style of Indian painting practiced in the Mithila region of Bihar and
the adjoining parts of Terai in Nepal. These paintings were done traditionally by the women on freshly
plastered mud walls and floors of huts but now they are also done on cloth, handmade paper and canvas.
o Cheriyal Scroll Painting: Cheriyal is a small village in Warangal district of Telangana state. Cheriyal
painting of the Deccan has also been classified as a form of painting for storytelling and to entertain the
audience along which it carries the rich cultural history and heritage with it. Cheriyal paintings are a
stylized form of nakashi art that are believed to be brought from Mughal emperors to south along with
them. These paintings are prevalent across the region and generally depict the scenes from Indian
mythological epics such as Ramayana, Mahabharata, Garuda purana, Krishna leela, Markandeya purana
etc.

Q 82.B
o Provision for reservation in matters of promotions for SC and ST‘s in state services was added by
77th Constitutional Amendment Act of 1995 . Hence correct answer is option (b)
o Right to Education under Article 21A of the Constitution was added by the 86th Constitutional
Amendment Act of 2002.
https://T.me/PDF4Exams

o Bodo, Dongri, Maithili and Santhali were added to Eighth Schedule by the 92nd Amendment Act of
2003.
o Right to form cooperatives societies under Article 19(1)(c) was added by 97th Constitutional
Amendment Act of 2011.

Q 83.D
o Pair 1 is correctly matched: The 24th Amendment Act, 1971 restored and extended the scope of power
of Parliament to amend the Constitution by adding the words ―amend by way of addition or variation or
repeal any provision in accordance with the provisions laid down in this Article‖ Further, the amendment
provided that ―Nothing in Article 13 shall apply to any amendment made under this article‖ by way of an
addition of Clause 3 to Article 368.This was upheld in Kesavananda Bharti V. State Of Kerela (AIR 1973
SC 1461).
o Pair 2 is correctly matched: 42nd amendment had made major changes. It included words such as
Socialist secular in the Preamble along with "integrity". It also made Parliament as Supreme Sovereign
Body envisaged to enact laws. It also encroached judiciary ,and restricted it to make judgement made
under DPSP. It laid down the Fundamental Duties of Indian citizens under Part IVA containing
article 51A of Indian Constitution. This amendment brought about the most widespread changes to the
Constitution in its history, and is sometimes called a "mini-Constitution".
 Pair 3 is correctly matched. The 61st Amendment of the Constitution of India lowered the voting age of
elections to the Lok Sabha and Legislative Assemblies of States from 21 years to 18 years.

Q 84.B
 Statement 1 is not correct and statement 2 is correct: The government has recently set up National
Water Informatics Centre (NWIC) to maintain a comprehensive water resources data. It would be
a repository of nation-wide water resources data and would work as a Subordinate Office under the
Ministry of Water Resources, River Development and Ganga Rejuvenation which is to be headed by
a Joint Secretary level officer.
 Statement 3 is not correct: NWIC will provide latest and reliable water data through web-based India
Water Resources Information System (India-WRIS) on a GIS platform in Public Domain.Thus, it will not
replace India-WRIS, rather use it for collecting information.
28 www.visionias.in ©Vision IAS

https://T.me/Civilsbuzz https://T.me/PDF4Exams
Downloaded From:- https://T.me/TestSeries4Exams

 Other features of NWIC -:


 It will also collaborate with leading national and international research institutes to provide technical
support to central and state organisations dealing with water emergency response of hydrological
extremes.
 It will provide a ‗Single Window‘ source of updated data on water resources & allied themes and
provide value added products and services to all stake holders for its management and sustainable
development.
 It is a component of National Hydrology Project and also in consonance with the National Water
Mission which has an objective of ―conservation of water, minimizing wastage and ensuring its more
equitable distribution through integrated water resources development and management‖.

Q 85.A
 The Constitution (under Article 15) prohibits discrimination against any citizen on grounds of
religion, race, caste, sex or place of birth and not on the ground of residence. This means that the
state can provide special benefits or give preference to its residents in matters that do not come within the
purview of the rights given by the Constitution to the Indian citizens. For example, a state may offer a
concession in fees for education to its residents. Any bill mandating such provision is not ultra-vires
https://T.me/PDF4Exams

and the governor will readily give his assent to the bill.

Q 86.B
 Statements 1 and 2 are correct: The rationale for introducing fundamental duties was to enable citizens
to become aware of their duties and responsibilities. As the fundamental rights were already part of the
constitution since inception, fundamental duties were introduced to achieve a better democratic balance
and strengthen democracy.
 Statement 3 is not correct: Fundamental duties were not introduced to promote socio-economic equality.
Directive principles of state policy are primarily aimed at socio-economic equality.

Q 87.B
 Fraternity means a sense of brotherhood.
 The Preamble declares that fraternity has to assure two things: the dignity of the individual and the unity
and integrity of the nation.
 The fundamental duties (Article 51A) say that it shall be the duty of every citizen of India to promote
harmony and the spirit of common brotherhood amongst all people of India transcending religious,
linguistic, regional or sectional diversities.
 The Constitution also promotes this feeling of fraternity through the system of single citizenship.
 The provisions under the Directive Principles of State Policy do not directly promote the feeling of
fraternity.

Q 88.A
o In a typical Semi-presidential system, the head of state is the president, directly elected by the people
with a large degree of power over the government, whilst the head of government is the prime minister
nominated by the president.
o It is also known as the French system. Unlike the parliamentary system, the president may have
significant day-to-day power.
o Example are France, Russia, South Korea etc. Hence pair 1 is correctly matched.
o An absolute monarchy is a form of monarchy in which one ruler has supreme authority and where that
authority is not restricted by any written laws, legislature, or customs. Example Saudi Arabia, Oman,
Qatar etc. Hence pair 2 is correctly matched.

29 www.visionias.in ©Vision IAS

https://T.me/Civilsbuzz https://T.me/PDF4Exams
Downloaded From:- https://T.me/TestSeries4Exams

o A parliamentary republic is a type of republic that operates under a parliamentary system of government
where the executive branch (the government) derives its legitimacy from and is accountable to the
legislature. Being a republic, it has an elected head either directly or indirectly.
o Since the United Kingdom has a hereditary, not elected, head, it is a constitutional monarchy. (with
Queen as head of state). However, it is a parliamentary democracy having parliament as the legislative
organ. Hence pair 3 is not correctly matched.
o Hence option (a) is the correct answer.

Q 89.A
 Statement 1 is correct and statement 2 is not correct: It is aimed at imparting green skills which are
required to adapt processes, services and products to climate change and the environmental rules and
necessities related to it. It is being implemented under the Environmental Information System (ENVIS)
scheme.
 GSDP endeavours to develop green skilled workers having technical knowledge and commitment to
sustainable development. GSDP will include process-based green skills such as, protecting ecosystems
and biodiversity, monitoring and managing activities such as waste, pollution, energy efficiency, impact
minimization and assessment etc. Thus, they will help in achieving -
https://T.me/PDF4Exams

 Intended Nationally Determined Contributions (INDCs) - It includes reduction of energy emission


intensity, shift to renewable sources of energy and creation of carbon sinks. GSDP will help in
achieving INDCs by protecting biodiversity (leading to creation of carbon sinks) and managing waste,
pollution and efficiency (leading to reduction of energy emission intensity).
 Sustainable Development Goals (SDGs) - It inlcudes goals related toelimination of poverty, huger,
education for all, climate action, gender equality etc. GSDP will help in achieving SDGs especially
SDG-14 which is related to urgent action to combat climate change and its impacts.
 National Biodiversity Targets (NBT) - It includes targets such as making the youth aware of values
of biodiversity, integration of values of biodiversity in planning processes, adoption of measures for
sustainable management of agriculture etc. GSDP would lead to skilling of youth in Green Skills
which would make them aware of the values of biodiversity. In addition, GSDP would lead to more
involvement of biodiversity values in planning process owing to training of experts in Green Skills.

Q 90.C
 The ‗Territory of India‘ includes states, union territories, and territories that may be acquired by the
Government of India at any future time. By declaring a UT as a state no new territorial area will be added,
hence no increase in territorial extent. Hence statement 1 is not correct.
 Being a sovereign state, India can acquire foreign territories, and increase 'Territory of India', according to
the modes recognised by international law:
 Cession (following treaty, purchase, gift, lease or plebiscite). Hence statement 3 is correct.
 Occupation (hitherto unoccupied by a recognised ruler)
 Conquest or subjugation. Hence statement 2 is correct.

Q 91.B
 The Multi-sectoral Development Programme (MsDP) has been restructured and renamed as Pradhan
Mantri Jan Vikas Karyakram (PMJVK).
 Pradhan Mantri Jan Vikas Karyakram (PMJVK) seeks to provide better socio-economic
infrastructure facilities to the minority communities particularly in the field of education, health &
skill development which would further lead to the lessening of the gap between the national average and
the minority communities with regard to backwardness parameters. Hence option (b) is the correct
answer.
 MsDP was identified as one of the Cores of the Core Schemes under National Development Agenda of
NITI Aayog.
30 www.visionias.in ©Vision IAS

https://T.me/Civilsbuzz https://T.me/PDF4Exams
Downloaded From:- https://T.me/TestSeries4Exams

 It is an initiative of the Ministry of Minority Affairs.


 The criteria for identification of Minority Concentration Towns (MCTs) and Cluster of Villages is based
on 2011 census:
 The population criteria for selection of cluster of villages has been lowered to 25% population of the
minority community (which was earlier at least 50%).
 Earlier only those Towns which were found backward in terms of both in Basic Amenities and Socio-
economic parameters were taken up as MCTs. Now, the Towns which were found backward in either
or both of the criteria have been taken up as MCT.
 Funding of the scheme would be from the budgetary provision of the Ministry of Minority Affairs.
The recurring/maintenance expenses will be borne by the State Government/UTs/Organization.
 80% would be earmarked for projects related to education, health and skill development.
 33 to 40% would be specifically allocated for women-centric projects.

Q 92.D
 Suva expert dialogue is an expert dialogue decided at COP23 (UNFCCC) in Bonn due to demands
of developing nations for a separate agenda item on loss and damage related with climate change.
 The dialogue aims for facilitating the mobilization and securing of expertise, and enhancement of support,
https://T.me/PDF4Exams

including finance, technology and capacity-building, for addressing loss and damage associated
with the adverse effects of climate change.
 Hence, option (d) is the correct answer.

Q 93.C
 S-400 Triumf is one of the world‘s most advanced air defence systems, developed by Russia.
 The system is a large complex of radars, control systems and different types of missiles.
 The highly automated S-400 has radars that can pick up an incoming object up to a 1,000 kilometres
away, track several dozen incoming objects simultaneously, distribute the targets to appropriate missile
systems and ensure a high success rate.
 The command post detects, tracks and identifies the target. Then the tracked object is taken over by
manned anti-aircraft missile systems of the complex, which launch the counter attack.
 It is in the middle of the ongoing stand-off between Russia and Western nations.
 Among the countries under pressure from the U.S. not to buy this weapon is India.

Q 94.D
 An amendment of the Constitution can be initiated only by the introduction of a bill for the purpose in
either House of Parliament and not in the state legislatures.
 The bill can be introduced either by a minister (even who is not a member of Parliament but only
maximum period of 6 months ) or by a private member ( Leader of Opposition ) and does not
require prior permission of the President.
 The Attorney General of India have the right to speak and take part in the proceedings of either House,
any joint sitting of both the Houses and any committee of Parliament of which he is a member, without
being entitled to vote. Thus, Attorney General cannot introduce a bill.

Q 95.B
 Indian citizenship can be acquired by birth, descent, registration and naturalization. The conditions and
procedure for acquisition of Indian citizenship as per the provision of the Citizenship Act, 1955 are given
below:
 By Birth: When a person is born in India, he is considered as citizen of India, subject to conditions
under the said act.

31 www.visionias.in ©Vision IAS

https://T.me/Civilsbuzz https://T.me/PDF4Exams
Downloaded From:- https://T.me/TestSeries4Exams

 By Descent: When a person is born outside India, but his parents are citizens of India, then that
person is considered as a citizen of India, subject to conditions under the said act.
 By Registration: Indian Citizenship by registration can be acquired by any person (not illegal
migrant), subject to conditions under the said act.
 By Naturalisation: The Central Government may grant a certificate of naturalization to any person
(not illegal migration), subject to certain qualifications like adequate knowledge of a language
specified in Eight Schedule of India, renounce his previous citizenship etc
 By Incorporation of Territory: Like, when Pondicherry became a part of India, the Government of
India issued the Citizenship (Pondicherry) Order, 1960 under Citizenship Act, 1955. People residing
in Pondicherry became citizens of India. Hence, this method is available with the Government of
India and not individual.

Q 96.D
 The Preamble reveals four ingredients or components:
o Source of authority of the Constitution: The Preamble states that the Constitution derives its
authority from the people of India.
o Nature of Indian State: It declares India to be of a sovereign, socialist, secular democratic and
republican polity.
https://T.me/PDF4Exams

o Objectives of the Constitution: It specifies justice, liberty, equality and fraternity as the objectives.
o Date of adoption of the Constitution: It stipulates November 26, 1949 as the date.

Q 97.A
 The Shanghai Cooperation Organisation (SCO) is a permanent intergovernmental international
organisation, formed on 15 June 2001 in Shanghai (China) by the Republic of Kazakhstan, the People's
Republic of China, the Kyrgyz Republic, the Russian Federation, the Republic of Tajikistan, and the
Republic of Uzbekistan.
 SCO comprises two regional bodies – SCO Secretariat and SCO-RATS (Regional Anti-Terrorism
Structure).
 As members of the SCO, countries are expected to be active in the SCO Secretariat and also
participate in the RATS, headquartered at Tashkent.
 SCO-RATS is a permanent body of the SCO and is intended to facilitate coordination and interaction
between competent authorities of the SCO Member States in the fight against terrorism, extremism and
separatism.
 A meeting of the defence ministers of the Shanghai Cooperation Organization (SCO) was held in Beijing
on April 24, 2018. This is the first meeting attended by India and Pakistan after they have become full-
fledged members of the Shanghai Cooperation Organization (SCO) at the Astana Summit on June 8-9,
2017.
 Counter-terrorism has been on the agenda of the SCO since its inception but has been boosted since
membership was granted to India and Pakistan in 2017.
 Its May 2018 meeting was hosted by Pakistan for the first time ever since it was granted membership.
India attended the meeting.

Q 98.C
o The Swachh Survekshan survey is conducted under Swachh Bharat Mission.
o The Ministry of Housing and Urban Affairs takes up the Swachh Survekshan in urban areas and the
Ministry of Drinking Water and Sanitation in rural areas. The Quality Council of India (QCI) is
responsible for carrying out the assessment. The Department of Industrial Policy & Promotion,
Ministry of Commerce & Industry, is the nodal ministry for QCI. Hence statement 1 is not correct.
o The 2018 survey captured the progress in following 6 broad parameters. Parameters & their respective
weights are given below:

32 www.visionias.in ©Vision IAS

https://T.me/Civilsbuzz https://T.me/PDF4Exams
Downloaded From:- https://T.me/TestSeries4Exams

 Collection and Transportation of Municipal Solid Waste - 30%. In 2017 it has the
maximum weight of 40%. Hence statement 2 is not correct.
 Processing and Disposal of Municipal Solid Waste - 25%
 Sanitation/ODF related progress - 30 %
 IEC (Information, Education and Communication) - 5%
 Capacity Building - 5%
 Innovation and Best Practices (Used for the first time to learn how our cities have responded the call
for Make India Clean and ODF by October 2019) - 5%
o Jharkhand has been adjudged the best-performing state, followed by Maharashtra and
Chhattisgarh. Hence statement 3 is correct.
o The three cleanest cities as per the Survekshan 2018 are Indore, Bhopal and Chandigarh.
o Hence option (c) is the correct answer.

Q 99.A
 Article 3 of the Indian Constitution authorises the Parliament to:
 form a new state by separation of territory from any state or by uniting two or more states or parts of
states or by uniting any territory to a part of any state;
https://T.me/PDF4Exams

 increase the area of any state;


 diminish the area of any state;
 alter the boundaries of any state; and
 alter the name of any state.
 The concerned State Legislature can express its views regarding the above to the President, but it is not
binding. Hence, the State Legislature has no power to enact the above provisions.

Q 100.D
 All statements are correct.
 The wearing and carrying of kirpans shall be deemed to be included in the profession of the Sikh religion.
In other words, Sikhs have the right to wear and carry kirpans (Article 25(2)).
 All minorities, whether based on religion or language, have the right to establish and administer
educational institutions of their choice. (Article 30(1)).
 No citizen can be denied admission into any educational institution maintained by the State or aided by
the State on grounds of religion, race, caste or language (Article 29(2)). Also, Article 21-A declares that
the State shall provide free and compulsory education to all children of the age of six to fourteen years in
such a manner as the State may determine.

Copyright © by Vision IAS


All rights are reserved. No part of this document may be reproduced, stored in a retrieval system or transmitted
in any form or by any means, electronic, mechanical, photocopying, recording or otherwise, without prior
permission of Vision IAS

33 www.visionias.in ©Vision IAS

https://T.me/Civilsbuzz https://T.me/PDF4Exams

You might also like